You are on page 1of 22

Neurology I Final Examination Review • Lesion B – clinically, patients complain of tunnel-like

COVERAGE vision, seen in cases of pituitary adenoma (pituitary tumor


I. Introduction to Clinical Neurosciences with size enough to impinge the optic chiasm)
II. Neuroanatomy • Lesion D – seen in cases of cerebrovascular disease
III. Neurologic History Taking and Examination involving M1 segment of the middle cerebral artery
IV. Neurolocalization (causing infarction of lateral geniculate nucleus)
V. Motor Weakness – Lower Motor Neuron System • Lesion E – loosely called “pie in the sky” and seen in
VI. Motor Weakness – Upper Motor Neuron System temporal lobe lesions (since the inferior optic radiations are
VII. Altered Sensorium and Syncope located in the temporal lobe)
VIII. Somatosensory Disorders
• Lesion F – seen in parietal lobe lesions (since the superior
IX. Dizziness and Dysequilibrium
optic radiations are located in the parietal lobe)
X. Epilepsy and Other Seizure Disorders
XI. Headache and Craniofacial Pain • Lesion G – similar to lesion D but there is characteristic
XII. Cerebrovascular Disease macular sparing (preservation of central vision) due to dual
Disclaimer and Note: These notes are compiled from different sources (lecturers, arterial supply of the macula from the middle cerebral
book, online peer-reviewed references) and are not intended to be used as the artery and posterior cerebral artery.
sole source for one’s review for the final examination. It must be used merely as
a guide as the student intends to reinforce previously learned knowledge and MOTOR PATHWAYS
integrate additional information from his/her reading in preparation for the
examination. May this review guide serve the student well as he/she intends to • In a patient presenting with motor dysfunction, tracing the
pass the subject with flying colors and appreciate the beauty of Neurology. anatomy of motor system from the upper motor neurons until
the lower motor neurons is critical. Remember the
INTRODUCTION TO CLINICAL NEUROSCIENCES Homunculus of Penfield since this a representation of the
• Diagnosis of neurological cases involve answering two key amount of innervation each body part receives in relation to
questions: (1) Where is the problem? (2) What is the its functional importance.
problem? (1) must always precede (2), not vice versa. • GENERALITIES:
• There are five pathways that the medical student must know: • (1) Corticobulbar = Corticonuclear. This tract supplies the
• (1) Visual Pathway head and neck, with the exception of the VAGUS NERVE
• (2) Motor Pathways: Corticospinal and Corticobulbar Tracts that supplies also the thoracic and abdominal areas.
• (3) Sensory Pathways: Dorsal Column – Medial Lemniscal • (2) Corticospinal tract supplies everything below the head
System (DCML) and Anterolateral/Spinothalamic Tract and neck.
(ALS/STT) • (3) Both the corticospinal and corticobulbar tracts CROSS
VISUAL (OPTIC) PATHWAY to the opposite side, only at different levels. All lesions
above the decussation cause CONTRALATERAL lesions.
• In a patient presenting with blurring of vision, thorough
All lesions below the decuessation cause IPSILATERAL
knowledge of the anatomy of the visual pathway guides the
lesions.
medical student in localizing lesions due to the unique
architecture of the visual pathway. • (4) The boundary between the upper and lower motor
neuron is the ANTERIOR HORN CELL.
• GENERALITIES:
• (1) TEMPORAL fibers of the optic nerve DO NOT CROSS
but NASAL fibers of the optic nerve CROSS to other
opposite side.
• (2) Optic NERVE -> Optic CHIASM -> Optic TRACT ->
LATERAL GENICULATE Nucleus -> Optic RADIATION ->
OCCIPITAL CORTEX. Do not interchange tract and
radiation.

Figure 2. Corticospinal and Corticobular Tracts


• Familiarize Figure 2 very well. Tracing the motor pathways
starts from the motor cortex to the corresponding muscles of
the head, neck, trunk, and upper and lower extremities. The
following is the pathway of the corticospinal tract:
• (1) Precentral gyrus = Primary Motor Cortex
• (2) Corona radiata – white matter projections
Figure 1. Visual Pathway and Corresponding Lesions • (3) Internal capsule
• Familiarize Figure 1 very well. Additional explanation include • (4) Midbrain – specifically the cerebral peduncles
the following: • (5) Pons – specifically the basis pontis
• Lesion A – also called monocular blindness, seen in • (6) Medulla – specifically the pyramids (decussation
cases of optic neuritis (inflammation of the optic nerve in occurs at this level, 85% becomes the lateral corticospinal
diseases such as Neuromyelitis Optica and Multiple tract whereas the remaining 15% that do not decussate
Sclerosis) becomes the anterior corticospinal tract)
• (7) Anterior horn cell – boundary between upper motor • (2) Dorsal Column Medial Lemniscal System is responsible
neuron and lower motor neuron system for transmission of fine touch, pressure, vibration and
• (8) Projections to the upper and lower extremities proprioception. Upon entry into the spinal cord, it ascends
• Terminologies: -paresis (= weakness), -plegia (= paralysis) IPSILATERALLY then DECUSSATES AT THE LEVEL OF
• In a patient manifesting with weakness or paralysis of MEDULLA to the opposite side.
specific parts of the body, there are certain generalities to • (3) First-order neurons generally start from the peripheral
help the medical student localize lesions. sensory receptors until the spinal cord. Second-order
• (1) Right or Left Lower Extremity Weakness – lesion is at neurons start from the spinal cord until the thalamus. Third-
the level of the spinal cord order neurons start from the thalamus until the
• (2) Bilateral Lower Extremity Weakness – lesion is at the somatosensory cortex.
level of the thoracic spinal cord
• (3) Bilateral Upper and Lower Extremity Weakness – lesion
is at the level of cervical spinal cord
• (4) Contralateral upper and lower extremity weakness with
Ipsilateral facial lesion – lesion at the level of the
brainstem (so called “crossed symptoms”)
• (5) Weakness on one side of the body – lesion is at the
level of the supratentorium (if with aphasia – probably
cortical lesion, if without aphasia – probably subcortical
lesion)
• Differentiation between upper and lower motor neuron
lesions is paramount and involves a thorough motor
examination during the neurologic examination. Remember
that these are GENERALITIES and there are always some
exceptions.
UMN Lesion LMN Lesion
Reflex Hyperreflexia Hypo/Areflexia
Tone Hypertonic/Spasticity Hypotonic/Flaccid
Babinski (+) (-)
Atrophy (-) (+)
Fasciculation (-) (+)
• Reflex and tone – the reason for an increase in these
components in UMN lesions is due to the loss of
modulation from the corticofugal fibers from the cortex. Figure 3. Anterolateral and Dorsal Column Medial Lemniscal
Think of the UMN as your typical strict parents such that Pathways
when they are out of town (“UMN lesion”), chaos results • Familiarize Figure 2 very well. Tracing the motor pathways
(“hyperreflexia and hypertonia”) starts from the motor cortex to the corresponding muscles of
• Babinski reflex – this is a pathologic reflex that manifests the head, neck, trunk, and upper and lower extremities. The
due to loss of inhibition of primitive reflexes by the frontal following is the pathway of the corticospinal tract:
lobe. In a neonate, infants and children up to 2 years of • (1) Peripheral sensory receptors
age, Babinski sign is normally seen. Beyond 2 years of age • (2) Dorsal root ganglion
and in adults, the presence of this reflex indicates an UMN • (3) Spinal cord – anterolateral system decussates while
lesion. Classically, this is seen in stroke, ALS and multiple dorsal column medial lemniscal system ascends
sclerosis, which all involve UMN lesions. On neurologic ipsillaterally
examination, this is elicited by rubbing a blunt instrument • (4) Brainstem – anterolateral continuously ascends
along the lateral side of the sole of foot from the heel then ipsilaterally while dorsal column medial lemniscal system
making a curve along the toes. decussates at the level of the medulla
• Atrophy and fasciculation – the reason for these two • (5) Thalamus – different thalamic nuclei receive the
phenomena in LMN lesions is due to loss of direct neural sensory fibers
stimulation of the muscles. Remember the muscles depend • (6) Postcentral gyrus = Primary Somatosensory Cortex
on the neuromuscular junction not just for its function but • Terminologies: -esthesia (sensation), -algesia (pain)
because the neural stimulation maintains these muscles.
Fasciculations are involuntary contractions or muscle NEUROANATOMY
twitches resulting from loss of voluntary control from the • Review of neuroanatomy will include the following:
LMN over its corresponding projections. • (1) Cerebral Cortex
• (2) Cranial Nerves
SENSORY PATHWAYS
• (3) Motor System
• In a patient presenting with sensory dysfunction, tracing the
• (4) Sensory System
anatomy of sensory system from the peripheral sensory
• (5) Reflexes
receptors until the sensory cortex is paramount. Remember
the Homunculus of Penfield since it represents the amount of • (6) Cerebellum
innervation a body part receives in relation to its functional • (7) Extrapyramidal Tracts and Basal Ganglia
importance. • (8) Cerebrovascular System
• GENERALITIES: CEREBRAL CORTEX
• (1) Anterolateral System is responsible for transmission of • Mainly responsible for awareness and requires an intact
pain, temperature and crude touch. Upon entry into the ARAS (located in the middle to upper pons) for wakefulness.
spinal cord, it IMMEDIATELY DECUSSATES to the • Divided into functional areas with correspondingly labeled
opposite side. Brodmann’s Area, the most important of which are the ff:
• Primary Somatosensory Cortex – BA 1,2,3 • Akinetic mutism: tending not to move/speak
• Primary Motor Cortex – BA 4 (“4 limbs”) • Saccades: fast eye movements
• Sensory Association Cortex – BA 5,7 • Nystagmus – involuntary, rapid and repetitive
• Pre-motor Area – BA 6 movements of the eyes
• Frontal Eye Field – BA 8 (“8 ~ ∞”) • FRONTAL LOBE lesions involve affectation of the (1) motor
• Primary Visual Cortex – BA 17 (“1+7 = 8 ~ ∞”) area, (2) prefrontal cortex, (3) olfactory nerve, (4) Broca’s
• Secondary Visual Cortex – BA 18,19 area, (5) parasagittal frontal lobe. Symptoms arise from
• Primary Auditory Cortex – BA 41 damage to these structures, such as the ff:
• Auditory Association Cortex – BA 42 UNILATERAL FRONTAL LOBE DISEASE: R or L
• Wernicke’s Area – BA 22 (1) Contralateral spastic hemiplegia and hemiparesis – due to
• Broca’s Area – BA 44,45 affectation of motor area
• 10% - Primary Areas, 80-90% - Association Areas (2) Symptoms of disinhibition (elevation of mood, increased
• Primary Area – direct connection to and from receptors talkativeness, tendency to joke inappropriately), lack of tact,
difficulty in adaptation and loss of initiative – due to affectation
• Secondary Area – makes sense of information from 1° areas
of prefrontal area
• Association Area – reception and analysis of information from
(3) Frontal release signs (grasp and suck reflexes) – due to
multiple cortical regions
affectation of the prefrontal area
• (1) Parieto-occipito-temporal – integrates information (4) Anosmia – due to affectation of olfactory tract in the area
from sensory, visual and auditory areas of the orbitofrontal lobe
• (2) Pre-frontal – plans complex patterns and movements *If entirely prefrontal area damage: no hemiplegia
• (3) Limbic - integrates behavior, motivation and emotions LEFT FRONTAL LOBE DISEASE
with sensory information (“Our brain tends to remember (1) Right-sided spastic hemiplegia and hemiparesis
memories that elicit human emotions”) (2) Broca’s aphasia, +/- apraxia of the lips and tongue – due to
• Special Function Areas: Wernicke’s and Broca’s Areas affectation of left lateral frontal lobe and motor area
(usually in the left hemisphere) innervating the lips and tongue (remember the Homunculus
• Wernicke’s – located in the superior temporal lobe, of Penfield from which the lips and tongue have the greatest
functions as interpretative area of language, lesion in this motor innervation)
area results to RECEPTIVE aphasia (inability to interpret (3) Sympathetic apraxia of the left hand – due to affectation of
written/spoken words) the dominant frontal lobe causing decreased transmission
• Broca’s – located in the left lateral frontal lobe, functions from the command center (the dominant frontal lobe) to the
for control of motor functions involved with speech, lesion in right frontal lobe (which controls the left hand)
this area results to EXPRESSIVE aphasia (inability to *Same as (2), (3), (4) from above
produce speech) RIGHT FRONTAL LOBE DISEASE
• Note: GLOBAL aphasia results if both Wernicke’s and (1) Left-sided spastic hemiplegia and hemiparesis
Broca’s areas are damaged. *Same as (2), (3), (4) from above
• There are 6 important lobes, and their functions are the ff: BILATERAL FRONTAL LOBE DISEASE
• Frontal Lobe – conscious thought-reasoning-planning, (1) Bilateral hemiparesis – due to affectation of bilateral motor
speech-movement, emotions-executive function, inhibition areas
of primitive reflexes, problem solving (2) Pseudobulbar palsy – paralysis, weakness and
• Parietal Lobe – perception-recognition-integration of hyperreflexia of the muscles in the face, mouth and pharyngeal
sensory stimuli area due to affectation of the corticobulbar tract from the
• Occipital Lobe – visual function and processing motor area (vs. bulbar palsy wherein there is direct affectation
• Temporal Lobe – smell, taste, sound perception- of the cranial nerve nuclei. You can think of this as the
recognition and integration, memory storage equivalent of LMN lesion in the spinal cord where anterior horn
• Insular Lobe (“Island of Reil”) – gustation cell = cranial nerve nuclei)
• Limbic Lobe – emotions, memory formation and learning (3) Abulia, akinetic mutism, inability to sustain attention and
solve problems, rigid thinking, bland affect, behavioral
LESIONS OF THE CEREBRAL CORTEX disinhibition, labile mood, frontal release signs, utilization
• Terminologies that are important to master are the ff: behavior – due to affectation of the prefrontal area
• -paresis: weakness, -plegia: paralysis, (4) Gait decomposition and sphincter incontinence – due to
• Palsy: uncontrollable tremors affectation of the parasagittal frontal lobe, which controls the
• Apraxia: inability to carry out motor activities on command lower extremities.
• Aphasia: inability to comprehend spoken/written words • PARIETAL LOBE lesions involve affectation of the (1)
• Ataxia: inability to coordinate voluntary movements sensory cortex, (2) sensory association areas, and (3)
• Agnosia: inability to recognize familiar objects superior loop of the optic radiation. This is where you will
• Prosopagnosia: inability to recognize faces observe unusual sensory symptoms and deficits. Symptoms
• Anosognosia: denial of neurologic deficit arise from damage of these structures, such as the ff:
• Dysnomia: impairment in recalling words/names UNILATERAL PARIETAL LOBE DISEASE: R or L
• Anosmia: inability to perceive odors (1) Corticosensory syndrome and contralateral sensory
• -anopsia: blindness in a visual field extinction – causes deficits in identifying more complex
• Achromatopsia: decreased light sensitivity, vision and color sensations such as direction, 2-point discrimination and
blindness shape that require sensory cortical integration whereas pain,
• Alexia: inability to understand written words temperature and touch are usually spared since this integration
• Dyslexia: impairment in reading and comprehension is already sufficient at the level of the thalamus
(2) Mild hemiparesis, hypotonia, poverty of movement and
• Dysmetria: under/overshoot of targeted movements
hemiataxia – due to affectation of the corticospinal tract (a
• Dysdiachokinesia: inability to perform rapid alternating
portion of which originates from the parietal lobe)
movements
(3) Inferior quadrantanopia – due to affectation of the superior
• Abulia: lack of willpower/initiative
loop of the optic radiations that pass through the parietal uncinate seizures
lobe (superior loop responsible for the inferior visual field) (3) Delirium and Disturbances in time perception
(4) Abolition of optokinetic nystagmus with target moving LEFT TEMPORAL LOBE DISEASE
towards contralateral side of the lesion – due to affectation of (1) Homonymous contralateral upper quadrantanopsia – due to
the contralateral parietal lobe (normally, repeated movement affectation of the inferior loop of the optic radiations
of the object and repeated detection will manifest as (2) Wernicke’s Aphasia – due to affectation of left superior
nystagmus and this is a physiologic response such as when temporal lobe
you are in a moving train looking outside at a stationary object; (3) Amusia – impairment in interpretation of music
with affectation of the parietal lobe, once an object moves (4) Impairment in tests of verbal material presented via the
towards the side opposite the affected parietal lobe, the object auditory sense
disappears and is no longer recognized by the brain and the (5) Dysnomia or amnesic aphasia – difficulty in recalling
normal optokinetic nystagmus can no longer be seen). names/words, probably due to affectation of hippocampus
(5) Contralateral hemineglect – due to affectation of the (6) Visual agnosia – inability to recognize visually-presented
parietal lobe (patients will tend to only recognize one side of objects (As you can observe, there are some exceptions and
the body with gaze preference towards the only side that is not all that is visual in deficit is purely occipital)
recognizable) (7) Korsakoff Syndrome – difficulty recalling and confabulates
LEFT PARIETAL LOBE DISEASE memories (confabulation as a compensation to make up for
(1) Alexia – due to affectation of the angular gyrus that is lost memory), probably due to affectation of hippocampus
located in the dominant parietal lobe, responsible for reading RIGHT TEMPORAL LOBE DISEASE
and writing (1) Homonymous contralateral upper quadrantanopsia
(2) Gerstmann Syndrome – manifests as a tetrad of symptoms (2) Impairment in tests of visually-presented nonverbal material
including Right-left disorientation (inability to discriminate left (3) Agnosia for sounds and some qualities of music
and right sides of the body), finger agnosia (inability to (4) Occasional inability to judge spatial relationships (Again,
recognize one’s own fingers or the examiner’s fingers), another exception where spatial is not purely parietal)
a/dyscalculia (impaired calculation), a/dysgraphia (impaired BILATERAL TEMPORAL LOBE DISEASE
writing), due to affectation of the dominant parietal lobe (1) Korsakoff amnesic defect – due to affectation of
(3) Tactile agnosia – inability to recognize objects placed in the hippocampus
hand (You can give the patient a familiar object such as a coin (2) Hypermetamorphosia – compulsion to attend to all visual
and they will not recognize it) stimuli
(4) Bilateral ideational and ideomotor apraxia – ideational (= (3) Apathy and Placidity
failure in conceptualization) apraxia is the inability to carry out (4) Kluver-Bucy Syndrome – due to affectation of the
functions that rely on semantic memory (eg. skill memory such amygdala and is characterized by fearless attitude and
as brushing teeth and combing hair) while ideomotor (= extreme curiosity about anything, rapid forgetfulness, places
breakdown in concept and performance) apraxia is the everything in the mouth and hypersexuality
inability to imitate actions but spontaneous actions remain • OCCIPITAL LOBE lesions involve affectation of the (1) visual
intact (eg. asking and demonstrating to a patient on how to cortices, and (2) visual association areas. Symptoms arise
comb one’s hair but he/she cannot do it on command but from damage of these structures, such as the ff:
without your command, the patient performs the task) UNILATERAL OCCIPITAL LOBE DISEASE: R or L
RIGHT PARIETAL LOBE DISEASE (1) Contralateral homonymous hemianopsia – with (peripheral)
(1) Predominantly hemineglect or without (central) macular sparing; if (+) macular sparing it is
(2) Visuospatial disorders – inability to perceive the usually seen in PCA stroke due to dual blood supply of the
relationship of objects in space to one another or to oneself macula from the PCA and MCA; if (-) macular sparing it is
(3) Topographic memory loss – inability to go from one place usually seen if the lesion at the level of ICA
to another, +/- map (2) Homonymous hemiachromatopsia
(4) Anosognosia, dressing and constructional apraxia – denial (3) Elementary (unformed) hallucinations – due to irritative
or unawareness of neurologic deficit and difficulties in lesions of the occipital cortex, more complex hallucinations are
performing complex tasks such as dressing, building, due to damage of the occipitoparietal and occipitotemporal
assembling and drawing objects, respectively. lobes
(5) Confusion, eyelid apraxia – tendency to keep eyes closed, LEFT OCCIPITAL LOBE DISEASE
resist lid opening and blepharospasm (1) Right-sided homonymous hemianopsia
BILATERAL PARIETAL LOBE DISEASE (2) Visual agnosia – due to affectation of the association
(1) Visual spatial imperceptions, spatial disorientation, areas (Remember that association areas serve highly
(2) Complete or Partial Balint’s Syndrome – the syndrome is specialized functions. Recognizing an object visually requires
secondary to affectation of the parietotemporal and parieto- not only vision but integration with its sensory meaning)
occipital areas, it is characterized by (1) spatial (3) Alexia and color-naming defect – due to affectation of deep
disorientation, (2) hemineglect, (3) optic apraxia (inability to white matter or splenium of the corpus callosum
move eyes on command but can spontaneously move eyes) (Remember that the corpus callosum is involved in connecting
and (4) simultagnosia (inability to perceive more than one the two cerebral hemispheres allowing the command center
object at a time, resulting in inability to perceive things as a from the dominant hemisphere to control the non-dominant
whole) hemisphere)
• TEMPORAL LOBE lesions involve affectation of the (1) RIGHT OCCIPITAL LOBE DISEASE
auditory cortex, (2) auditory association areas, (3) Wernicke’s (1) Left-sided homonymous hemianopsia
area, (4) hippocampal formation, (5) amygdala and (6) (2) Topographic memory loss and loss of visual orientation –
inferior loop of the optic radiation. Symptoms arise from remember that in right-sided parietal lobe lesions, these
damage of these structures, such as the ff: symptoms also emerge and this is probably due to affectation
UNILATERAL TEMPORAL LOBE DISEASE: R or L of the right occipitoparietal area
(1) Auditory, visual, olfactory and gustatory hallucinations (3) Visual illusions (metamorphosia) and hallucinations – due
(2) Dreamy state – due to affectation of the uncus seen in to extensive occipital lobe lesions with hallucinations more
common in the R > L occipital lobe have already decussated and contralateral LE weakness
BILATERAL OCCIPITAL LOBE DISEASE since fibers for the LE have not yet decussated)
(1) Cortical blindness but intact pupillary reflex – due to • Note that the corticospinal tract is influenced by the basal
sparing of the oculomotor nucleus since its nucleus is ganglia involves in fine tuning of movements, as well as the
located in the midbrain (Edinger-Westphal Nucleus) cerebellum and inhibitory interneurons involved in
(2) Anton Syndrome (visual anosognosia) – denial of blindness coordination of movements.
(3) Achromatopsia – loss of color perception • Remember the generalities and differentiation between UMN
(4) Prosopagnosia – loss facial recognition due to affectation and LMN. An additional pathologic reflex (aside from
of the occipitotemporal area, specifically the fusiform gyrus Babinski reflex) that we can observe in UMN lesions
(5) Simultagnosia – due to affectation of occipitoparietal area especially at the level of the cervical spine is the Hoffman
(6) Balint’s Syndrome – due to affectation of bilateral dorsal reflex. Lesions at lower spinal cord levels usually manifest
high occipitoparietal area with only Babinski reflex.
*Visual hallucinations with added objections/information usually
SENSORY SYSTEM
involves temporal lobe disease while visual distortions usually
involve occipital lobe disease • Dermatomes are not innervated by a single spinal nerve
• Limbic system – responsible for emotions, an important part (considerable overlap exists)
of memory formation and learning. It is composed of the ff: • DCML is responsible for position and vibration sense, 2-
• (1) Orbitofrontal gyrus – area above the olfactory tract point discrimination, fine touch, pressure
• (2) Cingulate gyrus – area above the corpus callosum • Terminates at the VPL nuclei of the thalamus. The VPL
nucleus is usually the main sensory thalamic nuclei, except
• (3) Subcallosal gyrus – aka Zuckerkandl’s gyrus
for VPM nuclei where taste and nerve fibers from the
• (4) Parahippocampal gyrus – area surrounding the
trigeminal nucleus terminate
hippocampal formation
• (From peripheral to central): Sensory receptors -> Dorsal
• Other parts: hippocampus (compared to the cerebral
root ganglion -> Nucleus Gracilis for LE (GRACE-ful LEGS)
cortex, contains only 3 histological layers), dentate gyrus,
and Nucleus Cuneatus for UE (C-unning HANDS) ->
supracallosal gyrus
Decussation at the level of medical lemniscus at upper
CRANIAL NERVES medulla -> pons (now called the internal arcuate fibers) ->
• CNs are can have sensory, motor or mixed functions. A pons -> midbrain -> VPL nuclei -> Parietal Lobe
st
mnemonic to remember is Some Say Money Matters But My • 1 -order neurons: sensory receptors to DRG
nd
Brother Says Big Brains Matter Most (from I – XII, S – • 2 -order neurons: DRG to VPL thalamic nucleus
rd
sensory, M – motor and B – both) • 3 -order nuclei: VPL thalamic nucleus to cerebral cortex
• Except for CN I and II, all the rest of the cranial nerves are • ALS/STT is responsible for crude touch, pain, temperature
located in the brainstem. • Anterior: for light touch and pressure, decussation at 2-3
• At the level of the MIDBRAIN lies CN III and IV levels above the first-order neuron
• Tectum – roof, Tegmentum – body, Cerebral peduncles • Lateral: for pain and temperature, decussation at same
– where the corticospinal tract passes through level of the spinal cord
• At the level of the superior colliculus (responsible for vision) • (From peripheral to central): Sensory receptors -> Dorsal
lies the oculomotor nuclei and mesencephalic nucleus root ganglion -> Decussation at the substantia gelatinosa
of CN V of spinal cord-> medulla (now called the spinothalamic
• At the level of the inferior colliculus (responsible for fibers) -> pons -> midbrain -> VPL nuclei -> Parietal Lobe
hearing) lies the trochlear nucleus. Note that CN IV is the • Lesions of the Sensory System:
only CN that decussates and exits dorsally in middle of the • Hypoesthesia, Anesthesia
superior and inferior colliculus in the dorsum of the • Paresthesia – spontaneous unpleasant, abnormal
midbrain to innervate the contralateral superior oblique sensations which are electrical in quality
muscle (ie. CN IV from the right part of the midbrain • Dysesthesia – same as paresthesia but provoked
innervates the left superior oblique, and vice versa) • Allodynia – non-noxious stimuli but perceived as painful (A
• At the level of the PONS lies CN V, VI and VII for Arte)
• CN V has the ff nuclei: mesencephalic (jaw position), • Hyperpathia – noxious stimuli but exaggeratedly perceived
primary sensory (pain and temperature sensation) and as painful
spinal
• CN VII wins around CN VI nuclei forming the facial CEREBELLUM
colliculus • 3 anatomical lobes exist, namely the anterior, posterior and
• At the level of the MEDULLA lies CN VIII, IX, X, XI and XII flocculonodular lobes
• CN X has the ff nuclei: dorsal nucleus (salivary glands, • Anterior lobe – superior surface, separated from posterior
heart and diaphragm), tractus solitarius (taste sensation lobe by primary fissure
formed by CN VII, IX and X) and ambiguus (innervation of • Posterior lobe – largest part
throat muscles, formed by CN IX and X) • Flocculonodular lobe – separates superior and inferior parts
of the cerebellum, separated from posterior lobe by
MOTOR SYSTEM posterolateral fissure
• Previously discussed but in addition, more lateral parts of the • 4 Nuclei (from medial to lateral) – Dentate, Emboliform,
Homunculus of Penfield are represented by the upper body Globose, Fastigial
structures while the more medial parts are represented by • Motor homunculus exists
the lower body structures • Vermis – coordination of axial body movements
• Primary motor tracts aka pyramidal tract and decussation • 3 function subdivisions:
occurs at the lower medulla (fibers for the UE are the first to • Archicerebellum (VESTIBULOcerebellum) –
decussate followed by fibers for the LE; hence, lesion in the flocculonodular lobe and fastigial nuclei, functions for eye
middle of the lower medulla can cause crossed weakness orientation during movement and coordination of head and
where there ipsilateral UE weakness since fibers for the UE limb position in different positions and while in motion,
lesion in this subdivision causes nystagmus due to lack of NEUROLOGIC EXAMINATION
eye orientation, constant head nodding due to lack of • Must be as systematic as possible: (1) Level of
head coordination with the body consciousness*, (2) MMSE, (3) Cranial Nerves, (4) Sensory,
• Paleocerebellum (SPINOcerebellum) – anterior lobe and (5) Motor, (6) Reflexes, (7) Cerebellum, (8) Gait and Balance,
globose and emboliform nuclei, functions for maintenance and other special examinations as indicated (Meningeal
of postural tone, lesion causes truncal ataxia due to loss signs, sweat pattern, Lasegue’s test, Tinel’s sign, Phalen’s
of postural tone maintenance maneuver).
• Neocerebellum (CEREBRO/PONTOcerebellum) – • Note that each examination must aide in the localization of
posterior lobe and dentate nuclei, functions for coordination lesions pointing to a disease that will explain the patient’s
and control and correction of movement, lesion causes clinical manifestations and phenomenology. This also
incoordination establishes the patient’s baseline neurologic status.
• Lesions: • Mini Mental Status Examination
• Nystagmus, action tremor, dysmetria, dysdiachokinesia, • Assess hearing and comprehension of simple conversation.
ataxia, titubation (rhythmic tremor of head and trunk), Allow patients to use hearing or visual aids if needed.
overshoot • A score of 0 is given if the person does not respond after
• Mnemonics (from our batchmate): The AFP went to the PAN THREE times.
pacific war. They provided service (SVC). They acquired • Orientation to time and place [10 pts] -> Registration [3 pts]
traumatic brain injury (TBI) and that’s the AND (End) of their -> Attention and Calculation [3 pts, serial sevens or world
lives. spelled backwards] -> Recall [3 pts] -> Naming [2 pts] ->
Lobe Phylogeny Functional Role Lesion
Repetition [1 pt] -> 3-stage command [3 pts] -> Reading [1
Anterior Paleo Spino Tone Ataxia (Gait) pt] -> Writing [1 pt] -> Copying [1 pt]
Flocculonodular Archi Vestibulo Balance Nystagmus • Lesions of the higher cortical systems result in abnormal
MMSE results.
Posterior Neo Cerebro Incoordination Dysmetria
• Cranial Nerves
• I (Olfactory) – note that patient’s eyes must be closed while
EXTRAPYRAMIDAL TRACTS AND BASAL GANGLIA wafting the bottle and irritating substances cannot be used
• Function: modulation of movement, supports voluntary as these will stimulate CN V
activity, principally controls proximal muscles • II (Optic) – note that this includes visual acuity, visual fields
• Caudate Nucleus + Putamen – Striatum (main receiver of by confrontation, pupillary reflex and fundoscopy.
afferent signals) • III (Oculomotor), IV (Trochlear) and VI (Abducens) – note
• Putamen + Globus Pallidus – Lentiform Nucleus that while making a wide H, observe for nystagmus during
• Direct pathway stimulates movement, Indirect pathway upward and lateral gaze; remember LRA-SOT.
inhibits movement • V (Trigeminal) – note that all subdivisions must be tested
• D1 receptors – stimulates movement (forehead for V1, cheeks for V2 and jaw for V3) for sensory
• D2 receptors – inhibits movement function (always compare both sides!), corneal/blink reflex
• Lesions: and jaw jerk reflex
• Altered muscle tone – rigidity/dystonia due to loss of • VII (Facial) – note that aside from facial movements, taste
coordination between contraction and relaxation of muscles is also tested
• Adventitious movements – tremors, chorea, athetosis, • VIII (Vestibulocochlear) – note that Weber is for
hemiballismus lateralization (can be placed on any bony prominence),
• Others: Masked expression, Glabellar Reflex (Myerson Rinne is for comparing air and bone conduction while
Sign) becomes evident, Gait Instability Schwabach is for comparing results with the examiner’s.
• IX (Glossopharyngeal) and X (Vagus) – note that examiner
NEUROLOGIC HISTORY TAKING AND EXAMINATION must observe the palatal elevation (ie. by saying “ah”) and
• Unique features of neurologic history taking include (1) the gag reflex (elevation of tongue and soft palate) and
handedness (remember that 95% of the population are right- pharyngeal constriction
handed), (2) address/residency (functions as a test of • XI (Spinal Accessory) – note that examiner must allow
memory and regional differences of diseases) patient to shrug shoulders or turn the head to each side first
• Timing in neurology is extremely critical (eg. stroke [acute before pushing the shoulders down
onset] versus tumors [chronic onset]) • XII (Hypoglossal) – note that examiner can ask patient to
push tongue against the inside of each cheek to test for
NEUROLOGIC HISTORY TAKING
strength.
• Common neurologic symptoms include (1) headache, (2)
• Sensory
dizziness, (3) weakness, (4) numbness, (5) loss of
• Always compare BOTH sides.
consciousness/altered sensorium, (6) seizures, (7) tremors
• For pain and temperature, usually testing either one of the
• Neurologic disorders can be classified into two types of brain
two is sufficient
injury: (1) focal (usually localizable), (2) diffuse (usually
• For vibration and proprioception, remember to place tuning
enough to cause cognitive impairment)
fork on the DIP and not the nailbed for vibration and
• The etiology of neurologic diseases can be localized based
demonstrate the positions of up and down before asking
on (1) structural (anatomical lesion) and (2) metabolic (eg.
patient to identify the position for proprioception.
infections, electrolyte imbalance)
• Do not test on calloused skin since it is insensitive.
• Motor
• Includes BULK, TONE, and STRENGTH.
• BULK: compare both sides and check if uni/bilateral and if
proximal/distal
• TONE: spasticity (velocity-dependent) versus rigidity
Figure 4. Etiologies based on types and timing of brain injury (velocity-independent), hypotonia
• STRENGTH: note that dominant side is slightly stronger Peripheral Nerve
and note/master muscle strength grading Step 3. Intersections
• Testing for muscle strength can start from distal to proximal - Combining the symptoms, their intersection or a location,
or vice versa, whichever is more convenient which can explain the appearance of all the symptoms is the
• Reflexes CORTEX.
• Note that patient must be completely RELAXED to elicit the - Therefore, lesion is most likely in the CORTEX.
deep tendon reflexes. Remember/master the grading of - Using the 3Ls, we have just levelized the lesion to the
reflexes. CORTICAL area, lateralized that the lesion is on the LEFT side
• Physiologic: Biceps (C5-C6), Triceps (C6-C7), (due to right-sided weakness, gaze deviation to the left), and
Brachioradialis (C5-C6), Patellar (L2-L4), Achilles (S1) likely localizing that the lesion is in the FRONTAL lobe area.
• Pathologic (seen in UMN lesions): Babinski and Babinski- - Therefore, we have a lesion in the LEFT FRONTAL
like reflexes, Ankle clonus, Glabellar tap, Rooting, Grasp, CORTEX. Due to the sudden nature of the symptoms and age
Palmomental, Tromner’s-Hoffman’s-Rossolimo’s of the patient, it is likely VASCULAR etiology.
• Cerebellum Answer: Ischemic Infarction of the Left Frontal Lobe secondary
• Requires integrated function of the motor system, sensory to Occlusion of the Superior Division of the MCA
system, vestibular system and cerebellar system to create
COORDINATION NEURAXIS LEVELS – NEUROLOGIC EXAMINATION
• Examinations include (1) Finger-to-nose test, (2) Heel-to- • Cortex – familiarize the anatomic lobes and their functions
shin test, (3) Rapid Alternating Movements, (4) Romberg’s • Brainstem – I and II not in the brainstem, III-IV in the
• Observe for dydiachokinesia, overshoot and nystagmus if midbrain, V-VII – pons, VIII-XII - medulla
one is suspecting cerebellar lesions. • Corticospinal tract – remember the generalities of
• Gait and Balance differentiating UMN and LMN injury
• Examinations include observation of fluidity of normal gait, • Note: most of the corticobulbar tracts are bilateral. Hence it
tandem gait, heel and toe walking. would take a severe lesion large enough to create complete
• Note that to distinguish vestibular and cerebellar lesions weakness of the innervations of the corticobulbar tract
presenting as balance problems, ask the patient to open • Central versus Peripheral Facial Palsy: due to contralateral
the eyes and if the balance problem lessens/disappears, innervation to the lower part of the face versus bilateral
then the problem is VESTIBULAR due to compensation of innervation to the upper part of the face. More commonly
the eyes but if the balance problem persists despite having observed is central facial palsy seen in stroke patients,
the eyes open, then the problem is CEREBELLAR hence why facial asymmetry is a warning sign of stroke.
• Special Examinations • CN XII is also preferentially contralaterally innervated.
• Meningeal signs: Brudzinski (B for “Batok”) and Kernig (K Table 1. Motor Loss Patterns
for “Kernig”) Site of Lesion Motor Loss
• Sweat pattern: for Horner’s syndrome Cortex, Corona Radiata Contralateral Hemiparesis
• Phalen’s Sign: for Carpal Tunnel Syndrome Internal Capsule Central Facial Palsy
Brainstem Contralateral Hemiparesis
NEUROLOCALIZATION Ipsilateral Cranial Nerve
• HISTORY/PE accurately localized most lesions while Deficits
damage to different divisions produce UNIQUE clinical Spinal Cord Quadri/Paraparesis
deficits. Cranial Nerve Sparing
• Main goal: to explain all S/Sx on the basis of ONE LESION. Anterior Horn Cell Localized/generalized
• 3Ls: LEVELIZE, LATERALIZE and LOCALIZE [WHERE?] -> weakness
Pathophysiology [HOW?] -> Pathology [WHAT? Answered by Nerve Root Loss follows root distribution
the neurologic history] Nerve Plexus Loss follows more than 1
• Steps: (1) List down all abnormal findings -> (2) nerve root distribution
Neuroanatomic Correlation -> (3) Intersections (may consider Polyneuropathy Hands-Arms, Feet-Legs
two or more lesions if no intersections) Mononeuropathy Single peripheral nerve
• Hard signs localizable while soft signs poorly localizable. NMJ Fatigable pattern
• Neurologic examination guides localization Muscle Proximal>Distal muscles
• MMSE – Cortex • Sensory – remember STT/ALS that decussates upon entry
• Cranial Nerve – Brainstem into the spinal cord while DCML that decussates at the upper
• Motor Exam and Reflexes – Corticospinal Tract medulla (higher level than the corticospinal tract)
• Gait and Coordination – Cerebellum • Central sensory nucleus of CN V extends from pons up to
the spinal cord, with lesion presenting as facial numbness
Example of Application of Steps in Neurolocalization
• CN V nuclei: Mesencephalic nucleus for proprioception,
A 70-year old patient comes in with sudden onset (3 hours
Main sensory nucleus for touch and pressure, and Spinal
prior) of slurring of speech, right-sided weakness (1/5 on the
nucleus for pain and temperature
arms and face, 3/5 on the lower limbs), gaze deviation to the
left, and right-sided facial drooping. Table 2. Sensory Loss Syndromes
Step 1. List down all abnormal findings Lesion Sensory Loss
- Slurring of speech, right-sided weakness, gaze deviation to Parietal Cortex Contralateral sensory loss
the left, right-sided facial drooping Thalamus Contralateral sensory loss
Step 2. Neuroanatomic Correlation Brainstem Ipsilateral facial sensory loss
- Slurring of speech: Cortex, Subcortex, Brainstem, Spinal Contateral limb sensory loss
Cord, Peripheral Nerves Cauda Equina Legs and Perineum
- Right-sided weakness (UE and face>LE): Cortex Nerve Root Dermatamol
- Gaze deviation to the left: Cortex, Subcortex, Brainstem All Peripheral Nerves Glove and stocking
- Right-sided facial drooping: Cortex, Subcortex, Brainstem, Single Peripheral Nerve Nerve distribution
• Cerebellum
Table 3. Sensory Loss Syndromes • Classic presentation is HANDST (hypotonia, assynergy of
Lesion Sensory Loss antagonist muscles, nystagmus,
Complete spinal cord injury All levels below the lesion dysmetria/dysdiachokinesia/dysarthria, stance and gait
Hemisection/Brown Sequard Ipsilateral – motor, vibration instability, tremor)
Syndrome and position sense • Neurologic examination (when cerebellum has a lesion):
Contralateral – pain and cerebellar deficits, motor dysfunction (intention > resting
temperature tremor, axial instability). The rest (higher cortical function,
Central Cord Syndrome Loss of pain and temperature cranial nerves, sensory, reflexes, pathologic reflexes) are
in band over segments normal.
occupied by the lesion • Spinal Cord
• Cerebellum: part of gait and coordination along with the • Classic presentation is weakness of distal/extensors >
motor, sensory systems and cortex proximal/flexors since corticospinal tract mainly controls
• Anterior lobe – regulates muscle tone, coordination of the distal extensors with sensory dysfunction at the
skilled voluntary movement – lesion causes gait ataxia specific spinal cord level, and bowel/bladder involvement
• Posterior lobe – for planning of voluntary movements – (retention first then incontinence)
lesion causes limb ataxia and hypotonia • Neurologic examination (when spinal cord has a lesion):
• Flocculonodular lobe – maintains balance, control of eye motor dysfunction (extensors>flexors, usually below the
movements – lesion causes titubation of head-trunk, lesion), sensory dysfunction (usually below the lesion),
drunken gait, truncal ataxia hyperreflexia, and pathologic reflexes. The rest (higher
• Visual System: lesions cause loss of vision in PREDICTABLE cortical function, cranial nerves, cerebellar) are ormal.
patterns • Root
• Gaze Deviation – pathways starts at BA 8 (Frontal Eye • Classic presentation is pain (sharp, stabbing, hot,
Field) -> internal capsule (anterior limb) -> decussates at radiating) with sensory/motor dysfunction at the
midbrain-pontine junction -> innervates PPRF -> innervates dermatomal/myotomal level since nerve roots contain a
CN VI and thereby lateral rectus muscle -> decussates mixture of sensory and motor nerve fibers
again back to the side of FEF activation via MLF -> • Neurologic examination (when nerve root has a lesion):
innervates medial rectus subnucleus of CN III and thereby motor/sensory dysfunction (usually asymmetric, severe if
medial rectus. Hence, right FEF allows gaze towards the more than 1 nerve root is transected, remember that
left and cortical lesions (above the decussation) cause dermatomal and myotomal levels usually overlap),
gaze deviation towards the ipsilateral side of the lesion hypo/areflexia, pathologic reflexes (Spurling’s, Dural
while brainstem lesions (below the decussation) cause Tension sign)
gaze deviation towards the contralateral side of the lesion • Peripheral Nerve
• Anisocoria – unequal pupillary sizes and must differentiate • Classic presentation is distal-predominant motor and
physiologic versus pathologic causes. Common sensory dysfunction (asymmetric) with possible
pathologic causes include Horner’s Syndrome and Third autonomic involvement and trophic changes (eg. skin, hair
Nerve Palsy. and nails).
• Neurologic examination (when peripheral nerve has a
ABNORMAL NEUROLOGIC FINDINGS lesion): distal motor/sensory dysfunction (atrophy,
• Analyze structures found in each level and relate these fasciculations, hypotonia, dysesthesias, hyperpathia,
structures to your neuroanatomy and neurophysiological allodynia), hypo/areflexia with possible cranial nerve
functions. deficits. The rest (higher cortical function, cerebellar,
• Cortical Brain pathologic reflexes) are normal.
• Classic presentation is abnormal higher cortical function • Neuromuscular Junction
(absence of which likely rules out cortical lesion) • Classic presentation is fatigability, proximal-symmetric
• Neurologic examination (when cortical brain has a lesion: weakness with preserved muscle bulk/tone and
abnormal higher cortical function, motor and sensory sensory function
dysfunction, hyperreflexia and pathologic reflexes, the • Neurologic examination (when NMJ has a lesion): fatigable
rest (cranial nerves, cerebellar) are normal. cranial nerve functions (ie. ptosis, dysconjugate gaze,
• Subcortical Brain slack jaw), motor dysfunction, normo/hypo/areflexia.
• Classic presentation is equal involvement of face/arm/leg The rest (higher cortical function, cerebellar, sensory,
(weakness and numbness) with visual field deficits pathologic reflexes) are normal.
(quadrantanopsia) • Muscle
• Neurologic examination (when subcortical brain has a • Classic presentation is proximal-symmetric UE-LE
lesion): cranial nerve deficit (visual field cuts), motor and weakness with cramping, aching, atrophy but
sensory dysfunction (equal involvement, Face=UE=LE), preserved sensation.
hyperreflexia and pathologic reflexes. The rest (higher • Neurologic examination (when muscle has a lesion):
cortical function and cerebellar) are normal. cranial nerve deficits (ie. ptosis, slack jaw, dysphagia,
• Brainstem dysarthria), motor dysfuncton, hyporeflexia (late in the
• Classic presentation is crossed deficits (ipsilateral cranial course of diseases). The rest (higher cortical function,
nerve deficits and contralateral motor/sensory dysfunction) cerebellar, sensory, pathologic reflexes) are normal.
• Neurologic examination (when brainstem has a lesion):
cranial nerve deficit (except for CN I and II; III-IV-VI: MOTOR WEAKNESS - LOWER MOTOR NEURON SYSTEM
Diplopia, V: Facial Numbness, VIII: Facial Drooping, IX-X- • Remember that the LMN starts from the anterior horn cells -
XII: Dysarthria and Dysphagia, XI: Decreased neck and > nerve root -> nerve plexus -> peripheral nerves -> NMJ ->
shoulder strength), motor and sensory dysfuntion, muscle
hyperreflexia, pathologic reflexes. The rest (higher
cortical function, cerebellar) are normal.
PATTERNS OF PNS LESIONS • NMJ: ocular (ie. ptosis and diplopia), bulbar (ie. dysphagia,
• Patterns are characterized based on (1) primary dysphonia), generalized (mainly PROXIMAL, can do
characteristic/hallmark symptom, (2) sensory pattern, (3) various tests during neurologic examination: hold out arms,
motor pattern, (4) reflexes, (5) atrophy, and (6) special look up, single breath counting test)
features • Muscle: proximal as seen in myositis (eg. Inclusion Body
• Based on HALLMARK/PRIMARY CHARACTERISTICS: Myositis)
Motor Neuron Disease UMN/LMN signs • Based on REFLEXES:
Nerve Root PAIN Motor Neuron Disease HYPER/HYPOREFLEXIA
Peripheral Nerve DISTAL>PROXIMAL Nerve Root MYOTOMAL
Neuromuscular Junction FATIGABILITY Peripheral Nerve DISTAL
Muscle PROXIMAL>DISTAL Neuromuscular Junction NORMAL
• Motor Neuron Disease (eg. ALS): affects distal and Muscle NORMAL (but if severe,
proximal muscles (ie. test distal muscles by asking patient hyporeflexia)
to WALK DOWN THE STAIRS, test proximal muscles by • Motor Neuron Disease: mixture of hyper/hyporeflexia due
asking patient to WALK UP THE STAIRS), usually starting to UMN and LMN lesions
out as ASYMMETRIC then becomes symmetric later on • Nerve Root: usually ASYMMETRIC, test the important
when affectation has become severe. reflexes (Biceps – C5/C6, Triceps – C6/C7, Brachioradialis
• Nerve Root (eg. Sciatica): causes pain plus myotomal – C5/C6, Patellar – L2/L3/L4, Ankle – S1/S2)
weakness and usually affects the cervical or lumbosacral • Peripheral Nerve: usually SYMMETRIC since metabolic
area since the area of C5-C7 and L5-S1 are the peak of problems will equally affect both sides
curvatures of the spine. • Based on ATROPHY:
• Peripheral Nerve (eg. Diabetic Polyneuropathy): distal Motor Neuron Disease Depending on which is WEAK
muscles affected first because it affects the longest Nerve Root MYOTOMAL
nerves first, which are found in the foot (ie. ask patient to Peripheral Nerve DISTAL
remove shoes to check for any atrophy) Neuromuscular Junction Normal
• NMJ (eg. Myasthenia Gravis): characteristic fatigability due Muscle PROXIMAL
to limited supply of neurotransmitters from the nerve PSEUDOHYPERTROPHY
terminal. Patients start out (during the day) having good • Pseudohypertrophy is the accumulation of fat and fibrous
muscle strength (5/5 upon testing) then declines (during the tissue that replaces atrophied muscle fibers.
late afternoon and night time).
• Muscle (eg. Polymyositis): affectation of the proximal • Based on SPECIAL FEATURES:
muscles maybe due to muscle bulk (in comparison to distal Motor Neuron Disease FASCICULATIONS
muscles) Nerve Root None
• Based on SENSORY PATTERN: Peripheral Nerve CONTRACTURES
Motor Neuron Disease NORMAL Neuromuscular Junction None
Nerve Root DERMATOMAL Muscle MUSCLE PAIN AND
Peripheral Nerve DISTAL>PROXIMAL TENDERNESS
Neuromuscular Junction NORMAL • Pseudohypertrophy is the accumulation of fat and fibrous
Muscle NORMAL tissue that replaces atrophied muscle fibers.
• Seen only in diseases that mainly affect the nerve root and • Tests used to diagnose different LMN lesions: EMG-NCV,
peripheral nerve. Why? It is because these components Spine X-ray/MRI (nerve root), Fasciculation Potentials
contain sensory fibers. Recall that nerve root and (Peripheral Nerve), Repetitive Nerve Stimulation (NMJ),
peripheral nerves contain a mixture of both motor and Blood Creatine Phosphokinase (Muscle)
sensory fibers.
• Nerve Root: presents as a DERMATOMAL distribution PERIPHERAL NEUROPATHY
(remember the important dermatomes: C5-C6-C7-C8-T1- • Entrapment Neuropathies include Carpal Tunnel Syndrome
T4-T10-L5-S4-S5). (Median Nerve), Cubital Tunnel Syndrome (Ulnar Nerve),
• Peripheral Nerve: presents usually in metabolic diseases Saturday Night Palsy (Radial Nerve), Lateral Femoral
such as DM and in a STOCKING-GLOVE distribution (ie. Cutaneous Nerve Injury, Common Peroneal Nerve Injury
numbness starts in the foot due to longer peripheral nerves • Carpal Tunnel Syndrome (Median Nerve): compression at
innervating the LE, usually VIBRATION first to be affected). wrist (ie. typing for long hours), confirmed by
• Based on MOTOR PATTERN: Phalen’s/Reverse Phalen’s/Tinel’s Sign, presents with
Motor Neuron Disease ASYMMETRIC (initially) atrophy of thenar muscles
Nerve Root MYOTOMAL • Cubital Tunnel Syndrome (Ulnar Nerve): compression at
Peripheral Nerve DISTAL elbow and occasionally wrist (Guyon’s Canal -> Ulnar
Neuromuscular Junction OCULAR, BULBAR, Tunnel Syndrome), confirmed by Tinel’s/Elbow Flexion,
GENERALIZED presents with atrophy of hypothenar eminence
Muscle PROXIMAL • Saturday Night Palsy (Radial Nerve): compression at spiral
• Motor Neuron Disease: Asymmetric and affects bulbar, groove of humerus, confirmed by Tinel’s, presents with wrist
upper and lower extremities drop
• Nerve Root: C5 (abduction, flexion), C6 (thumb abduction), • Meralgia Paresthetica (Lateral Femoral Cutaneous Nerve):
C7 (extension), C8 (digital abduction), L4 (knee extension), compression as the nerve passes under the inguinal
L5 (dorsiflexion, HEEL WALK), S1 (plantarflexion, TOE ligament, no motor presentation
WALK) • Common Peroneal Nerve Injury: compression at fibular
• Peripheral Nerve: weakness develops in specific head, confirmed with steppage gait, presents with foot drop.
innervations then progresses to contractures (eg. Papal
Benediction sign in ulnar nerve neuropahy)
MOTOR WEAKNESS - UPPER MOTOR NEURON SYSTEM extensor trunk-limb reflexes (LATERAL) critical for
• Proper reporting of the motor system: eg. Weakness is maintenance of proper posture; lesion results to motion
spastic, muscles are hypertonic, hyperreflexic with no sickness and vertigo.
fasciculations and atrophy • TECTOspinal (tectum in the midbrain to spinal cord):
• 4 components of the motor system: (1) direct activation facilitates coordinated head-eye movements
pathway, (2) control circuits, (3) indirect activation pathway,
FINAL COMMON PATHWAY
and (4) final common pathway.
• This is simply the pathway from peripheral nerve to muscle.
DIRECT ACTIVATION PATHWAYS Motor unit consists of LMN from the anterior horn cells in the
• Function: to effect voluntary activity, especially skilled spinal cord until the muscle. Neurotransmitter involves in
conscious movements of the fingers acetylcholine.
• Most pyramidal system neuronal cell bodies in the precentral
EVALUATION OF MOTOR WEAKNESS
gyrus and anterior part of the paracentral lobule (but
• UMN is commonly susceptible to Vascular, Demyelinating
some also in supplementary areas and parietal lobe)
and Traumatic insults. Evaluation involves determining the
• All axons from the motor cortex coverage toward the
onset, time curse and distribution of weakness and other
POSTERIOR LIMB of the internal capsule arranged from
associated neurologic findings. Proper reporting of findings
anterior posterior in the order of face, arm, leg, bladder and
during motor system testing is critical.
rectum
• Common cause of focal weakness: CVD, brain tumors,
• Tracts originate from the pyramidal cells in layer V of the
myelopathies, multiple sclerosis.
cerebral cortex, and a few from giant pyramidal cells of Betz
• The most frequent pyramidal tract disorder is a CAPSULAR
• Major transmitters include glutamate and GABA.
stroke referring to a stroke of internal capsule where the
CORTICOBULBAR TRACT corticospinal and corticorubral tracts converge and pass
• Usually the NONOCULAR brainstem LMNs are supplied through producing symptoms of paralysis of the contralateral
BILATERALLY by the motor area except for lower facial lower face, upper and lower extremities.
muscles, which are supplied solely by the contralateral • Temporal Pattern of Neurologic Symptoms
motor area. • Within minutes or less -> trauma or vascular etiology
• Clinical correlation: central (commonly seen in MCA stroke) • Over hours to days -> acute or subacute disorders such
and peripheral facial palsy (commonly seen in Bell’s Palsy) as cord compression, myelitis, GBS
• Bell’s Palsy: transient LMN lesion of CN VII believed to be • Over weeks to months -> subacute or chronic disorders
caused by a viral infection (eg. HSV) occurring at or beyond such as inherited polyneuropathies, myasthenia gravis,
the stylomastoid foramen resulting in the ff. symptoms: tumors, myelopathies
marked facial asymmetry, atrophy of facial muscles, facial • Fluctuates from day to day -> metabolic myopathies and
drooping (eyebrows, mouth), uncontrolled tearing, multiple sclerosis
inability to close the eyes (remember CN VII is the efferent • Fluctuates over course of the day -> myasthenia gravis
limb of the corneal reflex), difficulty keeping food in mouth • Anatomic Weakness Pattern
(remember that CN VII also controls some muscles of • PROXIMAL weakness -> difficulty in reaching upward,
mastication), ipsilateral hyperacusis (remember that CN VII going up the stairs
innervates the stapedius muscle) • DISTAL weakness -> difficulty in movements requiring fine
dexterity such as holding an object, writing, buttoning
CONTROL CIRCUITS
• BULBAR weakness -> cranial nerve deficits
• Function: to control and modify motor activity through the
• Do a systematic and complete neurologic examination to
CEREBELLUM and BASAL GANGLIA
guide your diagnostic approach (eg. MRI, vascular studies,
• Receives input from cerebral cortex and project back via the
lumbar puncture, EEG)
thalamus; Sends information to brainstem and indirect
• Spinal lesions commonly result in a sacral sparing
activation pathways
phenomenon due to somatotopic localization. Complete
• Basal ganglia: maintains background support/posture
transection results in functional abnormalities below the
needed for voluntary motor activity, lesions cause
lesion (voluntary movements, sensations, reflexes).
hypo/hyper/dyskinesia and rigidity
• Level of transection
• Cerebellum: coordinates/corrects movement errors of
• C5 above: respiratory difficulty
muscles, lesion cause defects in rate, range and force of
• C7 or above: quadriplegia
movements
• C8-T1: paraplegia and weakness of the hands
INDIRECT ACTIVATION PATHWAYS • Thoracic and Lumbar: paraplegia
• Function: mediate the automatic activities involved in • *Acute cervical/high thoracic lesions (usually trauma):
normal motor function (eg. maintaining proper posture) via sympathetic/neurogenic shock (transient hypotension,
subconscious coordination miosis, bradycardia but more permanent incontinence and
• Lesions usually result in abnormal muscle tone and reflexes impotence)
• 4 important pathways: • Lesion in the area of decussation: weakness of
• RUBROspinal (red nucleus to spinal cord): facilitates flexor contralateral upper extremity-ipsilateral lower extremity
movements of the contralateral upper extremity (CRUCIATE paralysis)
• RETICULOspinal (reticular formation to spinal cord): • Remember the mnemonic VITAMIN CDEF for differential
facilitates sensorimotor integration, postural reflexes diagnosis.
(pontine reticulospinal), and inhibition of segmental
ALTERED SENSORIUM AND SYNCOPE
muscle stretch and flexor reflexes (medullary
reticulospinal) • Consciousness is a state of full awareness of the self and
• VESTIBULOspinal (vestibular nuclei to spinal cord): one’s relationship to the environment.
facilitates coordinated control of head-eyes-neck • Sensorium is our awareness. Comatose is state of
movement for equilibrium (MEDIAL), and facilitates incapability to be aroused by stimuli or inner need
COMA AND ALTERED SENSORIUM mechanisms for respiratory control in the lower brainstem,
• State of consciousness controlled by the ARAS spanning the lesion most likely at the lower midbrain-upper pontine
brainstem reticular formation projecting to the thalamus, tegmentum
hypothalamus and cortex. Hence, we can surmise that a • APNEUSIS (pause of 2-3 seconds in full inspiration):
comatose patient has a lesion in the BRAINSTEM LEVEL caused by basilar artery occlusion, lesion most likely at the
OR ABOVE. lower pons
• Neurotransmitters involved include acetylcholine • CLUSTER BREATHING (few rapid deep breaths
(pedunculopontine tegmental and laterodorsal tegmental alternating with apneic episodes): lesion most likely at the
nuclei), norepinephrine (locus ceruleus), serotonin (dorsal lower pons
and median raphe nuclei), dopamine (ventral periaqueductal • ATAXIC/BIOT’S BREATHING (irregular breath patterns
grey matter) and histamine (tuberomammillary nucleus) with variable rate and depth): lesion most likely at the
• Cause is usually STRUCTURAL (asymmetric, focal findings dorsomedial medulla
with abnormal reflexes) versus METABOLIC (symmetric and • Motor
non-focal findings with normal reflexes • Examiner must do manual muscle testing, reflexes and
pathologic reflexes.
EVALUATION, EXAMINATION AND APPROACH TO COMA • Classic findings include clasp knife rigidity, (+) Babinski
• Remember S-P-E-R-M: Sensorium – Pupils – EOMs – • GLASGOW COMA SCALE: lowest score – 3, highest – 15
Respiration - Motor
• E4-V5-M6, important to master GCS scoring!
• Levels of Consciousness: elicited by using visual, auditory
• Known Causes of Coma include STRUCTURAL,
and noxious stimuli
METABOLIC and PHYSIOLOGIC causes
• AWAKE: oriented to time, place and person
• Structural: TBI, SAH, ICH, CVD Infarct, Tumor, Infection
• DROWSY (aka SOMNOLENT/LETHARGIC): dozes off
• Metabolic: Hypoxia, Hypercarbia, Fluid and Electrolyte
while responding to stimuli but can still follow simple
Disorders, Endocrine Abnormalities, Toxins, Vitamin
commands
Deficiency, requires good neurologic examination and EEG
• OBTUNDED (aka mentally dulled): decreased interest in
• Physiologic: Non-convulsive seizures, requires STAT EEG
surroundings, slow responses, short arousal periods,
• *Lab tests: blood tests, ABG, urinalysis, CSF analysis
unable to follow any commands
• If cause if Unknown, examiner may opt for a cranial CT/MRI
• STUPOROUS: responds with noxious stimuli by
(if abnormal – likely structural; if normal – likely metabolic,
grimacing/drawing away
hence do EEG)
• COMATOSE: no response to any stimuli, no verbal output,
• Monitoring acute organ injuries as likely causes of coma is
with either decorticate/decerebrate posturing
critical
• Pupillary Reflex (from superior to inferior)
• Why do EEG monitoring even when known structural and
• SMALL, REACTIVE: lesion possibly at diencephalon, but
metabolic causes, and known seizures have been controlled?
most commonly due to diffuse effects of drugs and
To rule out subclinical seizures which do not present with
metabolic encephalopathy
convulsions since these can cause coma.
• ANISOCORIA: lesion at CN III, commonly caused by uncal
herniation SYNCOPE
• MIDPOSITION, FIXED: lesion at midbrain • TRANSIENT and RAPID loss of consciousness with inability
• PINPOINT: lesion at pons to maintain postural tone followed by SPONTANEOUS
• EOMs: elicited by the oculocephalic reflex and cold calorics RECOVERY
• Oculocephalic (Doll’s Eye) reflex: normally inhibited and is • Most common causes include cardiac (arrhythmia,
released from inhibition during coma, results in conjugate structural), non-cardiac (vasovagal, orthostatic,
movement of the eyes in the opposite direction, which is cerebrovascular). This must be differentiated against non-
elicited by turning/tilting the head. Afferent arm from the syncopal attacks
labyrinth, vestibular nerve and neck proprioceptors. • CARDIAC: (1) arrhythmias (more common during exertion or
Efferent arm from the CN III-IV-VI and EOMs. Absence of supine, with palpitations at the time of syncope), (2)
the doll’s eye reflex in a comatose patient likely points to a structural; evaluation includes complete Hx/PE and 12-L
lesion in the area of the afferent and efferent arms. ECG
• Cold Calorics: normally, cold water causes slow phase • NON-CARDIAC: (1) nerually mediated (caused by
towards the ipsilateral ear and a corrective fast phase inappropriate vagal/sympathetic tone, with characteristic
towards the opposite ear, this is lost in comatose patients. prodrome of increased warmth/cold, fatigue,
Remember that only the SLOW PHASE IS lightheadedness, sweating and nausea, precipitated by
LOCALIZABLE. Note that COWS is not a very good tool stress, trauma, pain, sight of blood and prolonged standing,
for localization. Absence of cold calorics in a comatose and rapid recovery with headache, fatigue and weakness,
patient likely points to a lesion in the area of pons, medulla, types further subdivided into carotid sinus, situational faint
CN III-IV-VI and CN VIII. and classic vasovagal), (2) orthostatic hypotension (caused
• *Other cranial nerves for localization value: corneal reflex by insufficient autonomics to counter the drop in cardiac
test (for CN V), grimace and response to pain (for CN VII), output during upright posture), (3) cerebrovascular (caused
gag reflex (for CN IX-X) by basilar stroke, but almost never the cause of true
• Respiratory (from superior to inferior, based on breathing fainting)
patterns) • NON-SYNCOPAL ATTACKS: categorized into disorders with
• CHEYNE-STOKES (waxing and waning episodes of loss of consciousness (metabolic, epilepsy, TIA, intoxication)
hyperpnea with apnea in between): caused by isolation of and without loss of consciousness (psychogenic, TIA)
brainstem respiratory centers from the cerebrum, lesion
EVALUATION, EXAMINATION AND APPROACH TO
most likely at the bilateral cerebral hemispheres
SYNCOPE
• CENTRAL NEUROGENIC HYPERVENTILATION
• History must include events before, at the onset, during, and
(increased rate and depth of respiration enough to cause
respiratory alkalosis): caused by release of reflex after the attack
• Events before: position (supine/sitting -> ARRHYTHMIA; • Positive sensory symptoms: dysesthesia, allodynia,
exertion -> STRUCTURAL), activity, situation, risk factors hyperpathia
(prolonged standing -> VASOVAGAL; coughing-urination- • Negative sensory symptoms: anesthesia, hypoesthesia
defecation -> SITUATION; head rotation-neck movements - • Nerve Proper
> CAROTID SINUS; sudden standing from lying position -> • Generalized polyneuropathy: axonal (metabolic, toxic),
ORTHOSTATIC), triggers demyelinating (GBS)
• Events at the onset: w/ prodrome -> NEURALLY- • Focal neuropathy: such as the previously mentioned
MEDIATED w/o prodrome -> ARRHYTHMIA, entrapment neuropathies
• Events during: falling quickly -> ARRHYTHMIA, kneeling • Multifocal neuropathy: bilateral and asymmetric, such a in
over -> VASOVAGAL, brief and w/ Hx of heart disease -> Leprosy (Hypopigmented patches d/t infectious neuritis)
ARRHYTHMIA, >4-5 mins -> SEIZURE, METABOLIC, • Nerve Root
VASOVAGAL, tonic-clonic before falling -> SEIZURE, • Remember the characteristic PAIN in nerve root problems
tonic-clonic after falling -> VASOVAGAL, tongue biting -> • Most commonly due to disc herniations in the cervical and
SEIZURE lumbosacral region, can also affect thoracic region (eg.
• Events after: palpitations or waking up feeling normal -> HZV lying dormant in the dorsal root ganglion)
ARRHYTHMIA, chest pain -> STRUCTURAL HEART • Cauda Equina Syndrome (prolapse of central disk at L4-L5
DISEASE, nausea/vomiting/weakness/fatigue -> and L5-S1 manifesting as bilateral leg pain/weakness, saddle
VASOVAGAL, incontinence -> SEIZURE, anesthesia, bowel/bladder symptoms, impotence)
headache/diplopia/hemiparesis -> TIA/STROKE • Versus. Conus Medullaris Involvement: Sacral Sensory
• Others: medications, prior episodes (if >1 over multiple Sparing
years -> BENIGN, if >1 over short period of time -> • Sensory Neuropathies
SERIOUS CAUSE and likely ARRHYTHMIA), family history
• Types: complete transection, tabetic syndrome
of unexplained sudden death, past history of cardiac
(posterior column lesion), Brown-Sequard syndrome
disease/seizures/DM/alcohol intoxication
(hemitransection), syringomyelic syndrome (cystic spinal
• Physical Examination: cord mass from C4-T4, causing weakness of hands and
• Vital signs: check BP while supine for 5 minutes then after cape-like numbness), posterior column syndrome
standing for 3 minutes, if SBP/DBP falls by >20/10 mmHg - (posterior spinal artery occlusion causing loss of position
> ORTHOSTATIC SYNCOPE sense, hypoesthesia, ataxia and asynergia), anterior
• Heart: check for signs of heart failure/pulmonary embolism spinal artery syndrome (causes loss of motor function
• Lungs: check for PMI and murmurs and loss of pain and temperature sensation)
• Neurologic Examination: r/o focal findings suggestive of
stroke DIZZINESS AND DYSEQUILIBRIUM
• Diagnostic tests: cardiac (needs work-up, ECG, 2D Echo,
Stress Test, Holter), neurologic (does not require work-up, VESTIBULAR SYSTEM
may do EEG, tilt table testing) • Remember the structures: bony and membranous labyrinth, 3
semicircular canals (for angular acceleration, gaze stability,
SOMATOSENSORY DISORDERS with crista ampullaris), otoliths (for linear acceleration,
postural stability, with maculae acusticae)
CENTRAL COMPONENTS • Vestibular pathways include lateral and medial
• Remember the pathway of the DCML and the somatotopic vestibulospinal tracts
localization of the nucleus/fasciculus gracilis (more medial) • Vestibular connections with other systems include the ff:
and nucleus fasciculus cuneatus (more lateral). Lesions • MLF – for vestibulo-ocular reflex – for eye movement
result in the following symptoms: • Cerebellum – for vestibulospinal reflex – for coordination
• Loss of position and movement sense, two-point and timing of movement
discrimination, astereognosis and agraphesthesia, (+) • Neck – for vestibule-collic reflex – associated with neck
Romberg’s sign movements
• Remember the pathway of the ALS, which includes the
spinothalamic tract, spinoreticular tract and DIZZINESS, VERTIGO, PRESYNCOPE, DYSEQUILIBRIUM,
spinomesencephalic tract, and the somatotopic localization of AND LIGHTHEADEDNESS
the fibers from the upper limbs and trunk (more medial) and • Categories of dizziness include vertigo (to and fro or up and
lower limbs (more lateral). Lesions result in the following down movement of the body, most commonly PERIPHERAL
symptoms: in origin), disequilibrium (veering to one side), presyncope
• Loss of pain, temperature and crude touch (faintness sensation), lightheadedness (giddiness,
• Both ALS and DCML terminate in the VPL nucleus and accompanies anxiety, usually IDIOPATHIC/PSYCHIATRIC)
posterior nuclear group in the thalamus. • Important characteristics in dizziness: timing, triggers and
telltelate signs (to rule out acute vestibular syndrome at risk
PERIPHERAL COMPONENTS for stroke)
• Receptor organs: converts physical and chemical changes in • Important to differentiate PERIPHERAL vs. CENTRAL
the environment into electrical impulses that can be causes of vertigo:
processed by the CNS (transduction) PERIPHERAL CENTRAL
• Skin receptors have two classes: free nerve endings (for Focal deficits (-) (+)
pain and temperature) and encapsulated end organs (for Deafness (+) (-)
fine touch, 2-pt discrimination, vibration and pressure sense
Tinnitus (+) (-)
• Nociceptors usually made up of A-delta (light touch, sharp
Nytagmus Fast phase is Fast phase
and pricking pain) and C fibers (slow and burning pain) opposite the towards opposite
• Mechanoreceptors usually made up of A-alpha (limb lesion, (+) fixation side, (-) fixation
proprioception), A-beta (joint capsule pressure, flutter, suppression suppression
vibration, pressure) Latency 1-40 s None
Duration Few minutes Persistent SEIZURE VERSUS EPILEPSY
Intensity Severe Mild • Causes vary by age group: neonates (hypoxic ischemic
Systemic Marked nausea Less intense encephalopathy, congenital malformations, infections),
Symptoms and vomiting nausea and infants (fever, infections, post-hemorrhagic sequelae),
vomiting childhood (genetic disorders, trauma, neoplasm), young
• Causes of PERIPHERAL Vertigo: adults (idiopathic, trauma, drug withdrawal), middle-aged
• BPPV: caused by detachment of otolithic crystals and adults (trauma, neoplasm, vascular disease), old-aged
reattachment to posterior semicircular canal (most (vascular disease, tumor, degenerative disease)
commonly), characterized by paroxysmal vertigo and • Most common in INFANTS and ELDERLY
nystagmus especially on CHANGES IN POSITION, lasts • Differentials for seizures: SYNCOPE, pseudoseizures,
less than a minute but recurs periodically, diagnose by DIX- breath holding spell, sleep disorders, movement disorders
HALLPIKE Maneuver • Provoked seizures: fever, sleep deprivation, hyponatremia,
• Meniere’s: caused by increase in endolymph volume and hypoglycemia, hypoxia, drug/alcohol withdrawal
distention of the endolymphatic system with rupture of • CONVULSIONS: most common manifestation of seizures
membranous labyrinth and dumping of potassium- • Partial – occur in focal brain regions (most commonly frontal
containing endolymph into perilymph, characterized by and temporal lobes)
recurrent attacks of vertigo, tinnitus, deafness for minutes • Simple Partial: caused by focal cortical discharge,
to hours with nausea/vomiting/tendency to fall towards characterized by motor, somatosensory, autonomic,
affected ear. psycho-cognitive symptoms, generally brief and (-) LOC
• Labyrinthitis: caused by bacterial/viral infection of the • Complex Partial: caused by focal discharge from temporal
superior part of the vestibular nerve trunk, characterized by lobe, characterized by behavioral arrest followed by
paroxysmal but prolonged (hours to days) single attack of staring, automatisms and postictal confusion, and (+) loss
vertigo of awareness
• Toxic Bilateral Vestibulopathy: caused by prolonged • Secondarily Generalized: caused by partial seizures
exposure to aminoglycosides and other ototoxic evolving to secondarily generalized seizures
substances, characterized by disequilibrium • Generalized – occur in bilateral cerebral hemispheres
• Causes of CENTRAL Vertigo: • Absence (Petit Mal): common in children, characterized by
• Acoustic Neuroma: caused by space-occupying lesion sudden and brief lapse of consciousness, motionless blank
presenting in the cerebellopontine angle, characterized by stares but (-) loss of postural tone, EEG shows 3-Hz
high-frequency deafness, tinnitus, mild chronic imbalance spike and wave pattern
associated with headache, CN V-VII-X palsies, and • Myoclonic: characterized by sudden and brief shock-like
ipsilateral ataxia contractions during wakefulness (physiologic myoclonus
• Labyrinthine Infarction: caused by occlusion of the AICA or hypnic jerks can occur during sleep), EEG shows
and labyrinthine division of the internal auditory artery, bilaterally synchronous spikes or polyspikes and
characterized by single abrupt attack of vertigo, nausea, waves
vomiting without tinnitus or hearing loss. • Clonic: characterized by muscle spasms and bilateral jerks
• Brainstem Vertigo: caused by vascular, demyelinating and followed by rapid relaxation, EEG shows rapid spikes
neoplastic lesions that affect the vestibular nuclei and its with slow waves
connections, characterized by marked nystagmus without • Tonic: characterized by sudden, brief but sustained
vertigo associated with cranial nerve, sensory and motor contraction of extensor axial muscles and limb muscles
system involvement with LOC, EEG shows rapid spikes
• Dysequilibrium usually d/t multisensory deficit, decreased • Atonic: mistaken for fainting/syncope, characterized by
visual acuity, peripheral neuropathy, Parkinson’s. sudden loss of postural muscle tone with great risk for
• Lightheadedness usually categorized as psychiatric or head injury, EEG shows brief and generalized spike and
idiopathic types. wave discharges followed by diffuse slow waves
• Tonic-Clonic (Grand Mal): most common seizure type
EVALUATION, EXAMINATION AND APPROACH
resulting from METABOLIC DERANGEMENTS,
• History: description and type of the dizziness, positional
characterized by initial motor signs (brief trunk flexion,
effects, otologic-neurologic-psychiatric symptoms
opening of mouth and eyelids, upward eye deviation) ->
• Physical Examination: Vital signs (for orthostasis), hearing, tonic phase (extension of back, neck, arms and legs, ictal
cardiac, neurologic (especially gait), Dix-Hallpike test cry d/t respiratory muscle spasm, tongue biting, dilated and
(master/familiarize how to do this test), Head Thrust test unreactive pupils, incontinence) -> clonic phase (flexor
(used to assess vestibule-ocular reflex), Calorics testing spasms, eye blinking, salivary frothing, cyanosis, prominent
• Results of the Dix-Hallpike Maneuver: autonomic signs) -> terminal phase (deep coma, entire
• Central lesion: pure vertical and downbeat nystagmus body is limp)
• Peripheral lesion horizontal and upbeat/torsional • Epilepsies
nystagmus (BPPV), unidirectional and horizontal • Localization to specific cortical lobes possible or can be
(labyrinthitis) generalized.
• Further testing to rule out other categorizes of dizziness and • Generalized: can be idiopathic (genetic in origin, with (+)
their causes. FHx, begins early in life and BENIGN PROGNOSIS),
EPILEPSY AND OTHER SEIZURE DISORDERS secondary (structural cause identified usualy stroke, tumor
• Seizure is a transient and reversible alteration of behavior or trauma and mental retardation may occur), cryptogenic
caused by a paroxysmal, abnormal and excessive neuronal (structural cause suspected but cannot be identified)
discharge, common manifestation of neurologic diseases • Pathogenesis: shift of normal balance of excitation and
• Epilepsy is two or more seizures not directly provoked by inhibition of the CNS (precipitated by hyperexcitable
intracranial infection, drug withdrawal, acute metabolic neurons, high GLUTAMINE, low GABA)
changes of fever.
EVALUATION, EXAMINATION AND APPROACH • Ruling out secondary causes, especially life-threatening
• History: onset, presence of aura, ictal manifestations, diurnal causes (eg. SAH described “worst headache of my life”)
variation, triggering factors (flickering lights, alcohol, • Red flags for headache include the following:
medications, stress, fever, sleep deprivation), frequency, • First-ever attack of headache
seizure-free periods, injuries, frequency of ER consults. • Sudden and explosive headaches
• SEIZURE vs. SYNCOPE • Associated eye signs
SEIZURE SYNCOPE • Fever, Altered mentation/level of consciousness
Injury Common Rare • Change in character of the headaches
Incontinence Common Rare • Prior conditions
Post-ictal confusion Common Rare • *The mnemonic SNOOPS is used: Systemic symptoms,
Headache Common Rare Neurologic symptoms, Onset, Old age, Prior history,
Focal deficit Occassional Rare Secondary illness
Relation to Posture (-) (+) • Headaches requiring consultation:
Prodrome Short long • Severe, or “worst headache of your life”
• Diagnosis: CLINICAL diagnosis, requires accurate • Subacute headache with increasing frequency and severity
eyewitness account, but EEG is the single most important • Aggravated by exertion, coughing, bending, coitus
diagnostic procedure for confirming the diagnosis of epilepsy, • Persistently on the same side
MRI to rule out secondary causes • Awakens patient at night or disrupts sleep
• Other tests: CBC, RBS, blood chemistry, ECG, creatine • Associated with fever, stiff neck, blurred vision, slurred
kinase speech, weakness and confusion
• Treatment: AED only if RECURRENT or until a SECOND • No response to standard treatment
SEIZURE occurs (can only prevent seizures and most • Disabling and interfering with quality of life and activities of
importantly, STATUS EPILEPTICUS), MONOTHERAPY is daily living.
the recommended choice and choice depends on type of
seizures/epilepsy syndrome MIGRAINE
• Tonic – Clonic: Carbamazepine, Phenytoin, Valproic • Begins in childhood/adolescence/early adulthood, more
Acid, Phenobarbital predominantly seen in women
• Clonic: Valproic Acid • Characteristic: periodic, predominantly unilateral, often
• Absence: Valproic Acid, Ethosuximide pulsatile/throbbing headaches.
• Myoclonic: Valproic Acid, Clonazepam • Pathophysiology: Spreading Cortical Depression (causing
• Atonic: Valproic Acid dysfunction of brainstem serotonergeric and noradrenergic
• Goal of therapy: complete seizure control, monotherapy pain pathways)
and minimal side effects • Diagnostic Criteria (ICHD-3) for Migraine WITHOUT AURA
• Start therapy: definite diagnosis, risk of recurrence, weight • (A) At least 5 attacks fulfilling Criteria B-D
risk/benefits for pregnany women • (B) Headaches lasting 4-72 hours
• Discontinue therapy: 3-5 years of complete control, normal • (C) At least 3 of the following 4 characteristics: unilateral,
neurologic examination and normal EEG pulsatile, moderate or severe pain, aggravation by routine
• Other interventions: epileptic surgery (for intractable physical activity
epilepsies), ketogenic and medium chain triglyceride diet • (D) During headache, at least one of the following: nausea,
(augments GABA effects but high in fat) vomiting or both, and photophobia and phonophobia
• STATUS EPILEPTICUS: recurrent generalized convulsions • Diagnostic Criteria (ICHD-3) for Migraine WITH AURA
at a frequency that prevents regaining of consciousness • (A) At least 2 attacks fulfilling Criteria B
in the interval between seizures, prolonged convulsive • (B) At least 3 of the following 4 characteristics: (1)
status (>5 mins) but by definition it is specified to be at 15-20 one/more of the following fully reversible aura symptoms
minutes. indicate focal cerebral cortical/brainstem dysfunction, (2) at
• Medications given: Diazepam, Phenytoin, Phenobarbital least 1 aura symptoms develops gradually over >/= 4
• If all else fails: ANESTHESIZE! minutes or >2 occur in succession, (3) No aura symptoms
last > 60 minutes, (4) H/A follows aura with a free interval of
HEADACHE AND CRANIOFACIAL PAIN <60 minutes.
• Approach to headache includes differentiating primary versus • Treatment: NSAIDs (mild/moderate attacks), Triptans (severe
secondary headache. Most common are the primary benign attacks)
types (90%).
• Important to note the pain-insensitive structures (includes CLUSTER HEADACHE
basically everything within the brain parenchyma from the • Predominant in men
ventricular ependymal, choroid plexus and pial veins) • Characteristic: cluster pattern attacks, consistently unilateral
orbital localization with nightly recurrence, between 1-2 hours
HEADACHE after onset of sleep or several times during the night
• Two types of headache include primary (no exogenous (nocturnal headache)
cause) and secondary (with exogenous cause). • Pathophysiology (mostly theoretical): dysfunction of CN V
• Primary types of headache include Migraine, Cluster with hypothalamic mechanism related to the characteristic
Headache, and Tension Headache. Secondary types of nightly recurrence
headache include those caused by trauma, vascular • Associated vasomotor phenomena (unilateral): bulging and
disorders, metabolic, infection and psychiatric disorders. pulsating temporal artery, unilateral ptosis-swelling and eyelid
EVALUATION, EXAMINATION AND APPROACH redness, miosis and conjunctival injection, tearing, nasal
congestion and rhinorrhea, facial flusing and sweating.
• History/PE is always critical. Cardinal features should include
• Treatment: ergotamine, triptan, verapamil
OLD CARTS.
TENSION HEADACHE (60 mins), neurology expertise (15 mins), neurosurgery
• Most common headache variety. More likely to occur in expertise (2 hours), monitored bed (3 hours)
middle age and coincides with anxiety, fatigue and
ISCHEMIC STROKE
depression
• Most common type of stroke with embolic strokes being the
• Characteristic: bilateral, with occipitonuchal, temporal, or
most common type of ischemic stroke
frontal predominance, or diffuse extension over the top of the
• Types: atherothrombotic (less sudden), embolic (sudden,
cranium, dull and aching/fullness/tightness/pressure; lasts for
longer periods of time usually cardioembolic in origin, most frequently involving the
MCA), lacunar (<1cm, involves small penetrating branch
• Pathophysiology: excessive contraction of craniocervical
arteries)
muscles and associated constriction of scalp arteries
• Treatment, analgesics (relief for brief periods only), massage, HEMORRHAGIC STROKE
meditation and biofeedback techniques • Classically presents with sudden onset of LOC, very severe
headache, vomiting, seizures along with the focal deficits.
OTHER PRIMARY HEADACHE SYNDROMES
• Types: intracerebral (most commonly affects putamen and
• Idiopathic Stabbing Headaches: similar to Cluster headache
internal capsule), subarachnoid (usually due to saccular
but without associated vasomotor phenomena
aneurysm rupture)
• Cold Stimulus Headache/Ice Cream Headache: short-lasting
moderate to severe pain in the palate and throat, bilateral CLINICAL STROKE SYNDROMES
headache and self-limited • Anterior Cerebral Artery – A1 usually well tolerated due to
• Benign Cough Headache: “bursting” pain usually in the adequate collateral flow from contralateral ACA. A2
frontal and occipital area, unilateral/bilateral, usually following affectation usually results in the following deficits:
coughing/sneezing • Contralateral weakness (LE>UE and face)
• Benign Exertional Headache: episodes of bilateral, pulsatile • Contralateral numbness (LE>UE and face)
headache triggered by physical exercise • Bladder Incontinence
• Coital Headache: dull and bilateral headache increasing as • Sympathetic apraxia and frontal release signs
sexual increases and may evolve to severe throbbing and • Middle Cerebral Artery – most common site for embolic
explosive headache at the time of orgasm. strokes, especially affecting the branching arteries supplying
• Occipital Neuralgia: paroxysmal pain in the area of the basal ganglia and internal capsule.
distribution of the greater and lesser occipital nerves • M1 affectation – Contralateral weakness (UE and face >
CEREBROVASCULAR DISEASE LE), Contralateral weakness of the lower half of the face,
• Stroke is a rapid loss of brain function due to blood supply Contralateral hemisensory loss (UE and face > LE) and
disturbance to the brain, persisting beyond 24 hours. Brain hemineglect, Global aphasia, contralateral homonymous
requires 50-55ml/100g brain tissue/min hemianopsia, gaze deviation towards side of the lesion
• TIA is a focal neurologic deficit lasting less than 24 hours • Superior division affectation – Contralateral weakness (UE
and face > LE), Contralateral weakness and numbness of
• EMERGENCY, PREVENTABLE, TREATABLE!
the lower half of the face, Contralateral hemisensory loss
• Warning signs: SUDDEN, FOCAL (weakness, numbness,
(UE and face > LE), Broca’s aphasia
loss of speech, loss of comprehension, slurring of speech,
• Inferior division affectation – Contralateral homonymous
loss of vision, headache, dizziness, loss of balance)
hemianopsia, Contralateral upper quadratonopsia,
• Basic pathophysiology: vascular occlusion -> loss of oxygen
Contralateral constructional apraxia, Wernicke’s aphasia
and glucose supply -> activation of phospholipases ->
• Posterior Cerebral Artery – different syndromes exist such
neuronal membrane function alteration -> excitotoxicity
as the ff:
• Diagnosis: establish TIME OF ICTUS, do a PLAIN CRANIAL
• Unilateral occipital affectation – homonymous hemianopsia
CT SCAN
• Bilateral occipital affectation – Anton’s Syndrome (bilateral
• Differential Diagnosis: r/o common stroke MIMICKERS
homonymous hemianopsia, visual anosognosia)
(seizures, tumor, vertigo, syncope, dementia, bell’s palsy,
• Weber syndrome – contralateral weakness of the
hypertensive encephalopathy, dementia), recognize common
stroke CHAMELEONS (movement disorders – basal ganglia extremities with ipsilateral CN III lesion (ptosis, dilated
stroke [chorea -> putamen, ballimus -> subthalamic nucleus, pupil)
athetosis -> globus pallidus; agitation and delirium – temporal • Vertebral Artery – includes PICA lesion (Lateral
lobe stroke; sensory symptoms – parietal lobe affectation; Medullary/Wallenberg Syndrome) manifesting with the ff:
cortical blindness – occipital lobe affectation) • Contralateral hemisensory loss
• After ruling out other DDx, differentiate stroke subtypes, • Ipsilateral Ataxia
check eligibility for thrombolytic therapy (if ischemic stroke) • Ipsilateral facial numbness
then screen out complications of stroke • Ipsilateral palatal, pharyngeal and vocal cord paralysis,
• Assessment: stabilize, monitor vital signs, general medical Singultus (persistent hiccups)
assessment (patient history, onset), neurologic examination • Persistent vertigo, nystagmus
(GCS, NIHSS), plain cranial CT Scan • Basilar Artery – mainly CROSSED SYMPTOMS d/t
• Therapy: 3-4.5 hours is the golden hour in acute stroke, goal brainstem affectation and includes several syndromes such
is to save the ischemic PENUMBRA (UMBRA is the core as the ff:
infarct while OLIGEMIA is an area that surrounds the • Top Basilar – (+) transient LOC, hemianopsia, bilateral
penumbra) via recanalization (through thrombolytic therapy ptosis, pupillary enlargement with preserved pupillary light
with rTPA) and neurovascular protection (physiologic and reflex
pharmacologic means) • Midbasilar (Locked-in Syndrome) – (-) LOC, quadriplegia,
• Critical values: blood flow <70% - oligemia, <40% - electrical CN IV-VI palsy
disturbances, <20% - potential disturbances, <10% - death • AICA (Lateral Pontine, Marie-Foix Syndrome) –
• Critical time periods: Door to Doctor (10 mins), to CT contralateral weakness and numbness, ipsilateral ataxia,
completion (25 mins), CT scan reading (45 mins), treatment ipsilateral facial numbness/weakness
SAMPLE QUESTIONS Question 7. A 65-year old patient comes to your clinic for
follow-up consult of a previous cerebrovascular disease that
Question 1. A 40-year old female comes to your clinic occurred years ago. On neurologic examination, you ask the
complaining of blurring of vision of the left eye since 6 months patient to speak a sentence but you observe exorbitant effort in
prior to consult. She has been having progressively trying to speak. This patient most likely had a cerebrovascular
worsening headache, irregular menstruation and galactorrhea infarct of which part of the cerebral cortex?
as well. Upon consult, she now has blurring of vision on the A. Superior Frontal Lobe
right eye, saying to you that she feels like she has tunnel-like B. Inferior Frontal Lobe
vision. What is correct term for the patient’s chief complaint? C. Superior Temporal Lobe
A. Biparietal Hemianopsia D. Inferior Temporal Lobe
Answer: B. This is Broca’s aphasia, characterized by partial loss of the ability to speak written/spoken words.
B. Bitemporal Hemianopsia Effortful speech is observed, with comprehension usually remaining intact.

C. Bitemporal Homonymous Hemianopsia


Question 8. A 12-year old boy with above average
D. Biparietal Homonymous Hemianopsia
Answer: B. This is a case of pituitary adenoma. Key word is “tunnel-like vision”. This is not usually stated by the intelligence, while playing with his friends near the staircase of
patient but a description that must be elicited by the clinician. Progressively worsening and chronic headache
strongly supports a neoplastic lesion while irregular menstruation and galactorrhea are symptoms of
their school, fell from the stairs. Head injury followed the fall but
hyperpituitarism caused by the overgrowth of functional cells of the pituitary gland. no consult was done for fear of missing classes. In the days
Question 2. You are seeing a 65-year old right-handed patient that followed, the boy became quite lazy and lost his sense of
who is complaining of weakness on the left side of his body. direction as described by his siblings. His grades in math have
On neurologic examination, you were able to elicit right-sided also surprisingly declined. The boy’s lesion can most likely be
facial weakness upon cranial nerve testing in addition to 2/5 localized to which lobe?
strength on the left upper and left lower extremities. Without A. Right Parietal
the aid of imaging studies, the lesion is likely localized to __? B. Left Parietal
A. Cerebral Cortex C. Right Frontal
B. Subcortex D. Left Frontal
Answer: B. This is a case of Gerstmann Syndrome, which usually occurs in clinical situation such as the one
C. Brainstem above. Recall that this syndrome is found if the dominant parietal lobe is damaged, whereby in 95% of the
population it is found in the left parietal cortex.
D. Spinal Cord
Answer: C. This is a lesion located in the brainstem due to the presentation of crossed symptoms.
Question 9. Aside from the above symptoms (refer to
Question 3. A stray bullet hits an innocent by-stander on her Question 8) described, which of the following is another
back, resulting in paralysis of the bilateral lower extremities. prominent symptom of this syndrome?
This is correctly termed as ____ and indicates a lesion in the A. Finger anosognosia
____? B. Simultagnosia
A. Paraplegia, Thoracic Spinal Cord C. Agraphesthesia
B. Paraparesis, Thoracic Spinal Cord D. Acalculia
Answer: D. Note that the tetrad of symptoms for Gertsmann Syndrome includes finger agnosia, right-left
C. Paraplegia, Cervical Spinal Cord disorientation, agraphia and acalcuiia. Simultagnosia is usually found in Balint’s Syndrome.

D. Paraparesis, Cervical Spinal Cord


Answer: A. Note that –plegia is paralysis while –paresis is weakness. Paraplegia Is weakness of the bilateral Question 10. A 59-year old woman who presented to the
lower extremities while quadriplegia is weakness of all extremities usually seen in cervical spinal cord lesions.
emergency room at a tertiary care center with a 3-day history
Question 4. The dorsal column medial lemniscal pathway of mental status changes. According to her accompanying
decussates at the level of the _____ located in the _____? spouse, 3 days prior, she had eaten out and had one episode
A. Medial Lemniscus, Upper Medulla of diarrhea. Since then, she had slowly become increasingly
B. Mediolateral Lemniscus, Upper Medulla confused, trouble finding the right words and has been using
C. Medial Lemniscus, Lower Medulla incorrect words in sentences. These were accompanied by
D. Mediolateral Lemniscus, Lower Medulla mild anorexia and frontal headache. Neurologic examination
Answer: A. There is no such thing as the mediolateral lemniscus. The corticospinal tracts decussate at the level of shows mild dysnomia and subsequent cranial MRI showed
the lower medulla, not the DCML.
lesions, which would most likely be localized to which lobe?
Question 5. You are seeing a 55-year old patient with multiple A. Frontal Lobe
hypopigemented patches distributed in the trunk and B. Parietal Lobe
extremities. There is associated loss of sensation but no C. Temporal Lobe
pruritus. On further history taking, you note that the patient has D. Occipital Lobe
lives in the tropics for decades. Your top differential diagnosis Answer: C. This is a case Herpes Simplex encephalitis, which most commonly affects the temporal lobe as
manifested by Wernicke’s aphasia present in the patient, which is located in the left superior temporal lobe.
for the patient’s case includes infection caused by ___?
A. Nematodes Question 11. Confusion, trouble finding the right words and
B. Virus use of incorrect words as seen in our patient (refer to question
C. Protozoa 10) is correctly termed as what type of neurologic deficit?
D. Bacteria A. Expressive Aphasia
Answer D. This is a classic case of tuberculoid leprosy. Viruses that can cause neurologic symptoms include B. Reciprocal Aphasia
herpes simplex virus, varicella, and rabies virus. Protozoa such as Naegleria and Acanthamoeba cause
meningoencephalitis. Nematodes are not known to cause neurologic symptoms. C. Fluent Aphasia
D. Non-fluent Aphasia
Question 6. A 30-year old patient comes to your clinic and is Answer: C. Remember Wernicke’s = fluent = receptive aphasia while Broca’s = non-fluent = expressive aphasia.
accompanied by a companion who states that he has seen the
patient sucking his thumb uncontrollably. On neurologic Question 12. Should our patient (refer to question 10) have a
examination, (+) grasp reflex is elicited. Where is the most severe lesion in the affected lobe, this would result in what
likely location of your patient’s lesion? characteristic visual field deficit?
A. Parietal Lobe A. Superior Quadrantonopsia
B. Frontal Lobe B. Inferior Quadrantanopsia
C. Temporal Lobe C. Bitemporal Hemianopsia
D. Occipital Lobe D. Homonymous Hemianopsia
Answer: B. This is because the inferior loop of the optic radiations course through the temporal lobe. A lesion in
Answer: B. Thumb sucking and (+) grasp reflexes are primitive reflexes normally seen in newborns. Maturation of the parietal lobe would manifest with superior quadrantonopsia since the superior loop of the optic radiations
the prefrontal cortex results in inhibition of these primitive reflexes/frontal release signs. course through the parietal lobe.
Question 13. Occipital lobe lesions do not usually cause Question 19. While doing your neurologic examination of a
affectation of macula, unless the lesion results in an occlusion patient, you ask her to put his arms on the side while standing
to the supply of the macula at the level of which important upright and thereafter closing his eyes. You notice swaying
blood vessel? enough to cause the patient fall on his right side. You ask the
A. External Carotid Artery patient to open his eyes, causing minimal swaying and thereby
B. Internal Carotid Artery prevented him from falling on his right side. The lesion of this
C. Middle Cerebral Artery patient is most likely localized to which system?
D. Posterior Cerebral Artery A. Dorsal Column Medial Lemniscal System
E. Both C and D are correct. B. Cerebellum
Answer: B. Lesions at the level of Internal Carotid Artery are sufficient to cause macular affectation since this is
enough to cause ischemia of the macula. C. Vestibular System
D. Anterolateral System
Question 14. The lower part of the face receives ___ Answer: C. This is Romberg’s Test, and with correction or prevention of fall upon opening of the eyes indicates
the correction by the eyes is sufficient to compensate for a dysfunctional vestibular system.
innervation from the motor cortex while the upper part of the
face receives _____ cortex. _____ lesions of CN VII result in Question 20. M.P. is a 50-year old patient who presented with
central facial palsy while ____ lesions of CN VII result in gradual onset of tremors, difficulty in initiating movement and
peripheral facial palsy. robot-like gait that started around 1 year PTC, which he
A. Unilateral, Bilateral; Supranuclear, Infranuclear ignored as simply normal signs of aging. He was previously a
B. Unilateral, Bilateral; Infranuclear, Supranuclear world-renowned boxing champion, but retired at the age of 40
C. Bilateral, Unilateral; Supranuclear, Infranuclear after consecutive losses. The lesion of M.P is localized to
D. Bilateral, Unilateral; Infranuclear, Supranuclear ____, which secretes dopamine, normally causing _____?
Answer: A. Recall very well the difference between central and peripheral facial palsy. Central = from the UMN,
Peripheral = from the LMN.
A. Basal Ganglia, stimulation of movement
B. Substantia Nigra Pars Compacta, stimulation of movement
Question 15. A 55-year old woman from Pampanga presents C. Substantia Nigra Para Reticularis, stimulation of movement
with a 1-month history of inward deviation of the eyeball. There D. Caudate Nucleus, inhibition of movement
is associated deafness, tinnitus, epistaxis and cervical Answer: B. This is a case of early-onset Parkinson’s Disease, termed as Dementia Frugilistica, caused by
repeated head trauma common in boxers.
lymphadenopathy. Given the inward deviation of the eyeball,
which cranial nerve is most likely affected? Question 21. A 60-year old widow presented with a 1-month
A. CN II onset of “slowness” and “lack of focus”. The children of the
B. CN III patient claims that the patient seems uninterested in daily living
C. CN IV and claims that the patient would rather sleep for the whole day
D. CN VI than spend time with them. Upon doing your MMSE testing, it
Answer: D. As you will learn later on, this is a case of nasopharyngeal carcinoma causing a specific CN VI palsy
due to affectation of the pathway of the abducents nerve. Inward deviation of the eyeballs means there is an
shows intact attention and concentration despite the patient
overfunctioning medial rectus and loss of function of the lateral rectus, which is innervated by CN VI. Remember
LRA, SOT as mnemonic for EOMs.
initially insisting that he does not know the answers to the
questions. Which of the following statements about the patient
Question 16. A 25-year old man was shot at his back while is TRUE?
being chased down the police. Upon consult at the ER, he A. The patient has dementia, and the MMSE was not done
presented with the weakness of his left leg, loss of vibration correctly. MMSE needs to be repeatedly done.
and position sense of his left leg, and loss of pain and B. The patient has dementia, and the MMSE was done
temperature sensation of his right leg. All of the following correctly. Start the patient on physical and occupational
systems are affected, EXCEPT? therapy programs.
A. Dorsal Column Medial Lemniscal System C. The patient does not have dementia, and the MMSE is
B. Anterolateral System expected to be normal.
C. Corticorubral Tract D. The patient does not have dementia, and the MMSE is
D. Corticospinal Tract expected be abnormal.
Answer: C. This is a classic case of Brown Sequard Syndrome, caused by hemitransection of the spinal cord. A Answer: C. This is a case of pseudodemntia, where predominantly depressive symptoms appear to cause mental
bullet hit this patient causing a left hemitransection of the thoracic spinal cord, causing ipsilateral weakness and slowness and lack of ability to focus and concentrate. This can be differentiated from true dementia by the MMSE,
loss of position and vibration sense since CST and DCML have not yet decussated while causing contralateral where demented patients would give wrong answers, have poor attention and concentration, and appear
contralateral loss of pain and temperature sense since ALS decussates immediately upon entry into the spinal indifferent/unconcerned.
cord.

Question 22. An aneurysm of the posterior cerebral artery


Question 17. A 62-year old woman came into the ER due to
commonly results in affectation of which cranial nerve?
sudden onset of right-sided weakness and left-sided “down”
A. CN II
and “out” eyeball associated with miosis and mydriasis. The
B. CN III
lesion of this woman is most likely localized to which area?
C. CN IV
A. Right Subcortex
D. CN VI
B. Left Internal Capsule Answer: B. CN III is located in the area adjacent to the posterior cerebral artery, thereby aneurysms causing
C. Left Midbrain compression of CN III cause ptosis, mydriasis and a characteristic “down and out” eyeball.

D. Left Pons Question 23. The correct description for dysesthesia:


Answer: C. This is a case of Weber Syndrome, which is caused by occlusion of posterior cerebral artery that
supplies the medial superior midbrain area where the corticospinal tract (causing the contralateral weakness) and A. Spontaneous unpleasant, abnormal sensations which are
CN III (causing down and out due to overfunctioning lateral rectus and superior oblique) are located.
electrical in quality
Question 18. The motor homunculus that exist in the B. Noxious stimuli but exaggeratedly perceived as painful
cerebellum indicates that the trunk, head and neck area are C. Non-noxious stimuli but perceived as painful
represented by the ___ while the extremities are represented D. Provoked unpleasant, abnormal sensations which are
by the ___. electrical in quality
Answer: D. Remember that A is paresthesia, as one would experience after crossing his/her legs for long minutes
A. Vermis, Cerebellar Hemisphere and hours. B is allodynia (Remember A for Arte), and C is hyperpathia. Dysesthesia is same as paresthesia but is
B. Vermis, Anterior Lobe provoked.

C. Cerebellar Hemisphere, Vermis


D. Anterior Lobe, Vermis
Answer: A. The vermis located in the most medial part of the cerebellum is responsible for coordinating
movements in the axis of the body while cerebellar hemispheres are responsible for coordination of the
extremities.
Question 24. Which of the following is the correct description Question 29. A 22-year old patient comes in for complaints of
for a post-stroke patient, who on manual muscle testing, high-grade fever, headache and difficulty sleeping for around 5
showed active movement against gravity but upon application days. Upon neurologic examination, you are able to elicit
of some resistance, the patient can no longer actively move the positive meningeal signs. Which of the following is the BEST
extremity being tested? step in diagnosing this patient’s disease?
A. 1 A. Complete Blood Count
B. 2 B. Lumbar Puncture and CSF Analysis
C. 3 C. Cranial MRI with Contrast Enhancement
D. 4 D. Toxicology Screen
Answer: C. Remember and master the muscle strength grading. 1 is a barely detectable trace of contraction, 2 is Answer: B. The patient is presenting with classic signs and symptoms of meningitis. Lumbar puncture with CSF
active movement of the body with gravity eliminated, and 4 is active movement against gravity and some analysis is the best diagnostic method to confirm meningitis and differentiate its specific type. Complete blood
resistance. count can aid in confirming an infection. Imaging studies such as cranial MRI can aid in assessing the extent of
the disease, which would normally show meningeal thickening. Toxicology screening is not usually indicated for
meningitis.
Question 25. Upon eliciting the patient’s reflexes, you obtain
the following results: Biceps reflex +, Triceps Reflex ++, Question 30. While doing your neurologic examination of a
Brachioradialis +, Patellar Reflex ++, Achilles Reflex ++. With patient, you ask her to put her arms on the side while standing
these results, we can localize the specific LMN lesion of our upright and thereafter closing her eyes. You notice swaying
patient in which nerve roots? enough to cause the patient fall on her left side. You ask the
A. C4, C5 patient to open her eyes, but the swaying persisted and the
B. C5, C6 patient still fell on her left side. The lesion of this patient is most
C. C6, C7 likely localized to which system?
D. C7, C8 A. Dorsal Column Medial Lemniscal System
Answer: C. Remember the specific roots corresponding to each reflex: C5-C6 for biceps and brachioradialis, C6-
C7 for triceps, L2-L3-L4 for patellar, and S1 for Achilles. B. Cerebellum
C. Vestibular System
Question 26. You are suspecting patient with an UMN lesion D. Anterolateral System
and upon neurologic examination, you are tasked to elicit the Answer: B. This is Romberg’s Test, and without correction or prevention of fall upon opening of the eyes indicates
the correction by the eyes is insufficient to compensate from a damaged cerebellum.
Babinski reflex. This reflex is correctly elicited by doing the
following: Question 31. A 40-year old woman presents with acute onset
A. Lateral side of the sole of the foot is rubbed with a blunt of inability to speak, left facial drooping, left arm and leg
instrument that runs from the heel along a curve to the toes. weakness, and right gaze preference. Where is this lesion
B. Medial side of the sole of the foot is rubbed with a blunt most likely localized?
instrument that runs from the heel along a curve to the toes. A. Cortex
C. Lateral side of the sole of the foot is rubbed with a blunt B. Subcortex
instrument that runs from the toes along a curve to the heel. C. Brainstem
D. Medial side of the sole of the foot is rubbed with a blunt D. Spinal Cord
instrument that runs from the toes along a curve to the heel. Answer: A. Apply the steps in neurolocalizaton. Using these steps, we can localize the lesion to the cortex since
there is abnormal cortical function due to inability to speak or Broca’s aphasia. This is likely an MCA stroke
Answer: A. Remember that this is correct method to elicit the Babinski reflex. Never use a sharp instrument as caused by occlusion of the superior division of the MCA, causing ischemia/infarction the right frontal lobe.
this can harm the patient.

Question 27. You are presented with a 30-year old male Question 32. A 40-year old patient comes in with acute onset
patient who presents with frequent falls on the right side of his of right-sided weakness. On neurologic examination, you were
body. Upon neurologic examination, you elicit that he has able to elicit 1/5 strength on the right side of face, right arm and
dysmetria and intention tremor on the right side, explaining his right leg, in addition to numbness of the entire right side and
tendency to fall on that side. Which of the following is the best (+) Babinski reflex.
method to test for a cerebellar lesion? A. Cortex
A. Finger to Nose Test B. Subcortex
B. Romberg’s Sign C. Brainstem
C. Tandem Gait D. Spinal Cord
Answer: B. Apply the steps in neurolocalizaton. Using these steps, we can localize the lesion to the subcortex due
D. Rapid Alternating Movements to equal affectation of the face, arm and legs.
Answer: A. The finger-to-nose test is the best test to test for a cerebellar lesion. Remember that a right-sided
dysmetria and intention tremor most likely indicates a right-sided cerebellar lesion. Romberg’s sign indicates a
multisensory deficit. Tandem gait tests mainly the motor system and cerebellar system. Rapid Alternating
Question 33. Patient W.S. is a 50-year old patient who came
Movements is used to elicit dysdiachokinesia. in due to right-sided numbness, tendency to fall on the left side
and left-sided facial numbness associated with dysarthria,
Question 28. A patient comes to your clinic due to complaints
dysphonia and vertigo. Without doing your neurologic
of weakness of his lower extremities. On neurologic
examination, where can this lesion be localized?
examination, you observe that he needs to lift his legs higher
than normal to be able to walk properly. This characteristic gait A. Cortex
is called ___ and is caused by foot drop due to loss of ___? B. Subcortex
C. Brainstem
A. Steppage Gait, Plantarflexion
D. Cerebellum
B. Waddling Gait, Knee Flexion Answer: C. Apply the steps in neurolocalizaton. Using these steps, we can localize the lesion to the brainstem due
C. Waddling Gait, Knee Extension to the crossed deficits, specifically in the medulla, a syndrome called Wallenberg Syndrome secondary to
occulusion of the posterior inferior cerebellar artery.
D. Steppage Gait, Dorsiflexion
Answer: D. The steppage gait is characterized by loss of dorsiflexion, and along with waddling gait, maybe one of
the earliest clinical signs of a motor neuron disease, such as ALS.
Question 34. Cerebellar lesions usually manifest with all of the
following symptoms, EXCEPT?
A. Decreased muscle tone
B. Nystagmus
C. Action tremor
D. Hemiparesis
Answer: D. Remember the mnemonic HANDST, which stands for hypotonia, asynergy of antagonist muscles,
nystagmus, dysmetria/dysarthria, stance and gait problems, and tremors (action/intention > resting).
Question 35. A 37-year old patient comes to your clinic for Question 41. A 40-year old male patient comes in complaining
long-standing shooting pain that radiates down his right leg. of weakness of right arm weakness followed by right leg
There is associated weakness of the bilateral extremities, weakness, left leg weakness and left arm weakness. There is
rendering him unable to stand and walk properly without now noted difficulty in swallowing. On neurologic examination,
assistance. Where can this lesion be localized? he has a hyporeflexic biceps and triceps reflex, and
A. Cortex hyperreflexic patellar and ankle reflexes, (+) Babinski sign.
B. Subcortex Where is the most likely location of the patient’s lesion?
C. Brainstem A. Motor Neuron
D. Spinal Cord B. Root
Answer: D. This is specifically a lesion probably in the lower thoracic to upper lumbar region of the spinal cord,
affecting nerve roots, hence causing the characteristic pain seen in radiculopathy. C. Peripheral Nerve
D. Nerve Plexus
Question 36. A 28-year old man is seen at the ER following a Answer: A. This is a case of motor neuron disease, commonly seen in ALS or Lou Gehrig’s Disease. Note the
hallmark of mixed upper and lower motor neuron signs, and also characteristic asymmetry on initial presentation
motorcycle crash accident occurring a few minutes PTC. You becoming symmetric when the disease becomes chronic in duration.

observe flexion of the upper extremities and extension of the


Question 42. A 45-year old patient comes to your clinic for
lower extremities. This is termed as ___ and indicates a lesion
long-standing shooting pain that radiates down his lower
___.
extremities. There is associated weakness and numbness of
A. Decorticate posturing, above the red nucleus
the bilateral extremities, rendering him unable to stand and
B. Decorticate posturing, below the red nucleus
walk properly. Where can this lesion be localized?
C. Decerebrate posturing, above the red nucleus
A. Root
D. Decerebrate posturing, below the red nucleus
Answer: A. Note that it is not strictly damage to the sparing of the red nucleus from the rubrospinal tract that B. Peripheral Nerve
results in this type of posturing since this results in flexion of the upper extermities predominantly. It also involves
the corticospinal tract subserving the flexors of the lower extremity, resulting in extension of the lower extremities.
C. Neuromuscular Junction
D. Muscle
Question 37. If the patient above (refer to Question 36) has Answer: A. Remember the hallmark of radiculopathy, which is PAIN. In addition, presence of numbness
automatically eliminated NMJ and muscle disorders as only root and peripheral nerve disorders present with
incomprehensible mumbles and eye opening only upon sensory loss.

application of pain stimuli, what is the GCS score of this


Question 43. A patient is found to have weakness of abduction
patient?
of the little finger on the right hand and reduced pain sensation
A. GCS 8 (E3 V2 M3)
on the palmar surface of the little finger of the right hand. This
B. GCS 7 (E2 V2 M3)
lesion is most likely localized to ___?
C. GCS 9 (E2 V2 M4)
A. Radial Nerve
D. GCS 6 (E2 V2 M2)
Answer: B. Master the component of the GCS. Remember that we must indicate the specific scores to each B. Ulnar Nerve
component rather than just the overall score. A GCS<8 indicates respiratory depression and this patient will likely
require intubation. C. Axillary Nerve
D. Median Nerve
Answer: B. This is a classic case of ulnar nerve mononeuropathy. Note that the ulnar nerve innervates the palmar
Question 38. A 40-year old female patient was seen at the ER and dorsal surface of the lateral 1/3 of the hand and innervates the abductor digiti minimi.

due to sudden onset of headache, vomiting, weakness of all


extremities and seizures. After a few minutes, the patient was Question 44. A 55-year old woman presents with eyelid
seen to have arms extended, wrists rotated, legs extended and drooping and diplopia. She complains that her arms become
feet internally rotated. This is termed as ___ and indicates a heavy when she crochets from long periods of time with relief
lesion ___ upon resting. There are no other associated symptoms. Where
can this lesion be localized?
A. Decorticate posturing, above the red nucleus
B. Decorticate posturing, below the red nucleus A. Root
C. Decerebrate posturing, above the red nucleus B. Peripheral Nerve
D. Decerebrate posturing, below the red nucleus C. Neuromuscular Junction
Answer: D. Note that removing the influence of the rubrospinal tract subserving flexion of the upper extremities D. Muscle
allows the other indirect activation pathways to dominate, resulting in extension of the arms, in addition to Answer: C. This is a classic case of ocular myasthenia gravis, with FATIGABILITY as the hallmark of NMJ
extension of the legs. disorders. Diagnostic maneuvers tests can include single breath counting test (abnormal if patient cannot count to
10 while holding his/her breath) and repetitive nerve stimulation.

Question 39. If the patient above (refer to Question 38) has no


verbal output and eye opening only upon application of pain Question 45. A 7-year old patient comes in to your clinic due
stimuli, what is the GCS score of this patient? to history of repeated falls, fatigue and inability to climb stairs
but with normal mentation and no other associated symptoms.
A. GCS 5 (E2 V1 M2)
This patient’s lesion is most likely localized to ___?
B. GCS 5 (E2 V0 M3)
C. GCS 4 (E2 V1 M1) A. Root
D. GCS 4 (E1 V1 M2) B. Peripheral Nerve
Answer: A. Master the component of the GCS. Remember that we must indicate the specific scores to each C. Neuromuscular Junction
component rather than just the overall score. A GCS<8 indicates respiratory depression and this patient will likely
require intubation. D. Muscle
Answer: D. Muscle. This is a case of Duchenne’s Muscular Dystrophy characterized by early-onset weakness of
the PROXIMAL than distal muscles hence causing inability to climb stairs (requires use of proximal thigh
Question 40. A 58-year old male patient comes with due to muscles). Examination can show pseudohypertrophy of the affected muscles, elevated creatine kinase levels and
(+) Gower’s Sign.
sudden onset weakness of the right arm and weakness in the
left lower extremity following trauma to the upper neck. What is Question 46. A 33-year old patient who works as an online
the explanation for this patient’s symptoms? transcriptionist is complaining of recurrent numbness and
A. There is a brainstem infarction resulting in crossed deficits. weakness of his hands. You tap the ventral surface of his wrist
B. The patient is lying because this pattern of weakness is not area and suddenly, the patient complains of “pins and needles”
possible theoretically and clinically. sensation. The sign you elicited is called ___?
C. There is a lesion in the area of decussation of the A. Phalen’s Sign
corticospinal tract secondary to trauma. B. Reverse Phalen’s Sign
D. There is a lesion in the area of decussation of the C. Prayer’s Sign
corticospinal tract secondary to infarction. D. Tinel’s Sign
Answer: C. This characteristic pattern of weakness is seen in selective injury to the superior cervical spinal cord Answer: D. Note that Phalen’s sign is elicited by asking the patient to hold out the arms, flexing the wrists and
secondary to trauma. This type of paralysis is known as CRUCIATE paralysis. letting the hands hang down. Reverse Phalen’s /Prayer’s sign is elicited by holding out the arms, extending the
wrist and hands hang up.
Question 47. A pregnant patient is now into the early third Question 53. A 55-year old patient presents with dizziness
trimester of her current pregnancy and comes to your clinic upon abrupt change in positions, with associated nausea and
complaining of numbness of the thighs. Which of the following vomiting during the episodes. She is currently on Betahistine
nerve is most likely affected? once daily to prevent or minimize the episodes of dizziness.
A. Anterior Femoral Cutaneous Nerve Which of the following is characteristic of this condition?
B. Lateral Femoral Cutaneous Nerve A. The attacks occur only occasionally.
C. Anterolateral Femoral Cutaneous Nerve B. There attacks are paroxysmal and each episode can last for
D. Common Peroneal Nerve more than 5 minutes.
Answer: C. This is a classic case of lateral femoral cutaneous nerve injury. Pregnancy increases abdominal girth
causing nerve compression of the nerve as it passes under the inguinal ligament. C. The attacks last only for seconds or less than a minute.
D. There is associated hearing loss on the affected side.
Question 48. A 21-year old male colleagiate wrestler presents Answer: C. BPPV is characterized by a paroxysmal vertigo and nystagmus especially on changes in position, with
attacks lasting LESS THAN A MINUTE or for only SECONDS, with no associated hearing loss. The attacks recur
to your clinic with left elbow pain, associated with decreased periodically for months to years.

sensation along the medial forearm and decreased motor


Question 54. A 58-year old patient comes in due to severe
function in his hands and wrist. The patient recalls partial loss
attacks of dizziness that last for days with associated nausea
of function following a direct below to the ___?
and vomiting, and tinnitus and deafness. The cause of the
A. Spiral Groove
patient’s vertigo is likely ___?
B. Cubital Tunnel
A. Peripheral
C. Guyon’s Canal
B. Central
D. Carpal Tunnel
Answer: B. This is a classic case of ulnar nerve neuropathy/cubital tunnel syndrome following direct trauma to the C. Insufficient information is given to conclude whether the
elbow, likely damaging the ulner nerve as it passes through the cubital tunnel.
lesion is peripheral or central in origin
Question 49. A 24-year old male patient was brought to the D. The presence of tinnitus and deafness makes a vestibular
ER after sustaining multiple physical injuries. Upon seeing the lesion unlikely.
Answer: A. Remember the table on differences between peripheral and central causes of vertigo. Nausea,
patient, you observe that he mostly responds with incoherent vomiting, tinnitus and deafness are characteristics of a peripheral cause of vertigo.

mumbles, doses off while responding to stimuli but still able to


Question 55. A 30-year old patient comes in due to deafness,
follow simple commands. How would you describe the patient’s
and prolonged balance problems. There is associated facial
level of consciousness?
asymmetry and ipsilateral ataxia. The cause of the patient’s
A. Lethargic
symptoms is likely ___?
B. Obtunded
A. Peripheral
C. Stuporous
B. Central
D. Comatose
Answer: A. Lethargy is characterized by the patient’ above plus disorganized movements. The important C. Insufficient information is given to conclude whether the
characteristic is still being able to follow simple commands, which if he is unable to do so, he would be classified
as obtunded.
lesion is peripheral or central in origin
D. The presence of deafness and balance problems makes a
Question 50. The same patient above (refer to Question 49) vestibular lesion unlikely.
was seen by the ER doctor on duty and saw that his pupils Answer: B. Remember the table on differences between peripheral and central causes of vertigo. While deafness
is a more commonly seen in peripheral causes of vertigo, the presence of focal neurologic deficits makes a
were pinpoint with barely perceptible reaction to strong light. central lesion more likely. Do an cranial MRI for this patient to rule out a cerebellopontine angle tumor.

Based on the pupillary reaction, the location of the lesion is


Question 56. The following somatosensory deficits are
most likely in the ___?
described as positive sensory deficits, EXCEPT?
A. Midbrain
A. Paresthesia
B. Pons
B. Dysesthesia
C. Diencpehalon
C. Allodynia
D. Non-localizable lesion. This is a metabolic encephalopathy.
Answer: B. Remember, pinpoint pupils = pontine lesion. D. Hypoesthesia
Answer: D. A, B and C are positive symptoms because they cause added sensations. Hypoesthesia causes
characteristic decreased sensation, which is negative symptom.
Question 51. You are seeing a patient with altered sensorium
and while eliciting the Doll’e eye reflex, you observe the Question 57. The correct definition for syncope is ___?
following: Head turn to the right is followed by eyes turning to A. Rapid onset of transient loss of consciousness with an
the left. Head turn to the left is followed by eyes turning to the inability to maintain postural tone followed by spontaneous
right. Head turned upwards is followed by eyes going down recovery
while head turned downward is followed by eyes going up. B. Rapid onset of permanent loss of consciousness with an
What can we infer from these results? inability to maintain postural tone followed by spontaneous
A. Brainstem is intact. recovery.
B. Midbrain lesion is likely. C. Rapid onset of transient loss of consciousness with an
C. Pontine lesion is likely. inability to maintain postural tone followed by delayed recovery.
D. Non-conclusive results D. Rapid onset of permanent loss of consciousness with an
Answer: A. The normally inhibited Doll’s eye reflex in conscious patients is lost when sensorium is altered,
especially in comatose patients. The results seen with the patient are typical for an intact Doll’s eye inability to maintain postural tone followed by delayed recovery.
reflex/oculocephalic reflex. Any deviation from this pattern indicates a brainstem lesion, where the CN subserving Answer: A. The closes differential diagnosis for syncope is seizures. Remember, it is RAPID ONSET,
the EOMs are located. TRANSIENT loss of consciousness, and SPONTANEOUS recovery.

Question 52. Which of the following is INCORRECT about the


Dix-Hallpike Test?
A. Patient is seated and turned to one side at 30 degrees.
B. Patient is then helped to recline rapidly so that the head
hangs over the table
C. Within several seconds, this test elicits vertigo and
nystagmus that is horizontal with a rotary component.
D. Frenzel goggles can be used to aid in observing for
nystagmus since this disables visual fixation that can suppress
nystagmus.
Answer: A. Other choices are correct. However, the head is turned to one side at 45 DEGREES, not 30 degrees.
This is used to align the posterior semicircular canal with the sagittal plane of the head.
Question 58. A 74-yeard old patient sought was brought to Question 62. A 28-year old female is referred to your clinic
consult at the ER after 2 episodes of fainting within 24 hours. because of odd behavior. She had an episode witnessed by
He has a past history of hypertension, dyslipidemia, diabetes her mother where she suddenly stood up, mumbled
mellitus, and previous CABG but no recurrent chest pains. He incoherently, and fell to the floor, had a few jerks, and with
was seen just sitting down while reading book during the noted urinary incontinence. She woke up a few minutes later
attacks. There was no nausea and vomiting, and diaphoresis. and could not recall what happened. Her neurologic
He appeared pale and was unconscious for about 1 minute examination is normal. Laboratory tests and imaging did not
with no movements of bowel/bladder dysfunction. He woke up reveal any abnormality. Based on the patient’s case above,
feeling normal. The rest of the history and physical examination what is the type of seizure?
were unremarkable. Based on the patient’s case, what is the A. Simple Partial
most likely diagnosis? B. Complex Partial
A. Cardiac Syncope – Electrical cause C. Secondarily Generalized
B. Cardiac Syncope – Structural cause D. Generalized
C. Orthostatic-mediated Syncope Answer: C. Note that the patient started with intact consciousness then subsequently collapsed and lost
consciousness, characteristic of secondarily generalized seizures
D. Vasovagal Syncope
Answer: A. The history is consistent with a cardiac syncope, specifically an electrical cause that can occur with Question 63. If the patient above (refer to Question 62) had
the patient at rest. If patient was seen doing exertion, this likely rules in a structural cause.
persistent odd behavior for an extended period of time, this
Question 59. Based on the patient’s case above (refer to would be termed as ___ and one consideration for this
Question 58), which of the following laboratory tests is condition is prolonged convulsive status for longer than ___?
absolutely critical for diagnosis of the patient’s cause of A. Status Epilepticus, 3 minutes
syncope? B. Status Epilepticus, 5 minutes
A. Complete Blood Count C. Status Epilepticus, 15 minutes
B. Stat EEG D. Status Epilepticus, 30 minutes
C. 12-L ECG Answer: B. The strict definition is 15-30 minutes. However, in the context of clinical practice, it is more practical to
consider 5 minutes as the threshold since this condition requires immediate treatment due to cardiorespiratory
D. 2D Echocardiography dysfunction, hyperthermia, metabolic derangements that can lead to irreversible neuronal injury.
Answer: C. Electrical causes point to rhythm abnormalities and a 12-L is absolutely critical for diagnosing the
specific type of dysrhythmia causing ineffective pumping action of the heart and subsequently causing decreased Question 64. Normal cerebral blood flow is ___ and critical
blood flow to the brain. If a structural cause is suspected, a 2D echocardiography can be done.
level of hypoperfusion that abolishes brain function leading to
Question 60. Based on the patient’s case above (refer to tissue damage is ___?
Question 58), what laboratory test can you request for to rule A. 50-55ml/100g brain tissue/minute, 10-12 ml/100g brain
out the closest differential diagnosis for the patient’s case? tissue/minute
A. Complete Blood Count B. 50-55ml/100g brain tissue/minute, 12-23 ml/100g brain
B. Stat EEG tissue/minute
C. 12-L ECG C. 60-65ml/100g brain tissue/minute, 10-12 ml/100g brain
D. 2D Echocardiography tissue/minute
Answer: B. Again, seizures are the closest differential diagnosis for syncope and EEG can be used to monitor for
possible subclinical or non-convulsive seizures D. 60-65ml/100g brain tissue/minute, 12-23 ml/100g brain
tissue/minute
Question 61. The correct definition for seizure is ___? Answer: B. These values are critical since anything less than the 12 ml/100g/minute can already lead to infarction.

A. Transient and irreversible alteration of behavior caused by a Question 65. The most common site for an intracerebral
paroxysmal, abnormal and excessive neuronal discharge from hemorrhage to occur is in the ___?
pathologically excitable neurons.
A. Putamen
B. Transient and reversible alteration of behavior caused by a
B. Subthalamic Nucleus
paroxysmal, abnormal and excessive neuronal discharge from
C. Internal Capsule
pathologically excitable neurons.
D. Cerebellum
C. Transient and irreversible alteration of behavior caused by a Answer: A.
occasional, abnormal and excessive neuronal discharge from
pathologically excitable neurons. Question 66. A 75-year old patient was brought to the ER after
D. Transient and reversible alteration of behavior caused by a sudden onset of right-sided weakness and numbness with
occasional, abnormal and excessive neuronal discharge from slurring of speech. He went to bed at 2300H and was seen with
pathologically excitable neurons. these symptoms at 0500H the following day. What is the time
Answer: B. This definition closely resembles that of syncope but there are indeed a few differences, especially in of ictus and time that thrombolytics can be given based on the
the pathophysiology of seizures
golden hour of stroke?
A. 2300H, thrombolytics must ideally be given at 0200H
Question 61. The correct definition for seizure is ___? B. 2300H, thrombolytics must ideally be given at 0330H
A. Transient and irreversible alteration of behavior caused by a C. 0500H, thrombolytics must ideally be given at 0800H
paroxysmal, abnormal and excessive neuronal discharge from D. 0500H, thrombolytics must ideally be given at 0930H
Answer: A. Note that the time of ictus is the time that the patient was last seen normal and not the time at which
pathologically excitable neurons. he/she woke up already manifesting with the aforementioned symptoms. Golden hour is ideally at 3 hours but it
B. Transient and reversible alteration of behavior caused by a can be stretched to 4.5 hours.

paroxysmal, abnormal and excessive neuronal discharge from Question 67. The patient above (refer to Question 68) has a
pathologically excitable neurons. stroke most probably localized to the MCA. What visual deficit
C. Transient and irreversible alteration of behavior caused by a can we expect from the patient given his other neurologic
occasional, abnormal and excessive neuronal discharge from symptoms?
pathologically excitable neurons. A. Inferior quadrantonopsia
D. Transient and reversible alteration of behavior caused by a B. Superior quadrantonopsia
occasional, abnormal and excessive neuronal discharge from C. Homonymous Hemianopsia
pathologically excitable neurons. D. Bitemporal Hemianopsia
Answer: B. This definition closely resembles that of syncope but there are indeed a few differences, especially in
Answer: A. Note and remember the visual pathway as it passes through the parietal lobe area, where the superior
the pathophysiology of seizures
loop of the optic radiations is located.
Question 68. The mnemonic “SNOOPS” can be used to
identify red flags for headache and the mnemonic covers all of
the following red flag symptoms, EXCEPT:
A. Fever and stiff neck
B. Sudden onset
C. Neurologic deficits
D. Recurrent but constant in intensity
Answer: D. recurrent but constant in intensity of headaches is not a red flag since this points towards a primary
benign etiology. However, if it is becoming more frequent and increasing in intensity, that is a red flag.

Question 69. The characteristics typical for migraine headache


include the ff:
A. Periodic, predominantly unilateral, often dull/aching in
quality
B. Periodic, predominantly unilateral, often dull/aching in
quality
C. Periodic, exclusively unilateral, often pulsatile/throbbing
headaches.
D. Periodic, predominantly unilateral, often pulsatile/throbbing
headaches
Answer: D. Note that it is indeed periodic and can be bilateral but predominantly unilateral, with a pulsatile and
throbbing quality of headaches.

Question 70. A 26-year old right-handed female presents to


the ER with acute onset of severe headache. She states that
on her way to work few hours PTC, she suddenly felt a severe
and sharp pain in her head followed by loss of consciousness.
There was associated nausea, vomiting and neck pain. Vital
signs are stable. Patient appears to be in discomfort with eyes
shut closed, and neurologic examination is unremarkable.
Based on the patient’s case above, what is the probable
diagnosis?
A. Complicated Migraine
B. Tension Headache
C. Brain Tumor
D. Subarachnoid Hemorrhage
Answer: D. Choice A would usually present with migraine associated with a transient neurologic deficit that
resolves within 72 hours. Choice B is usually associated with stress but the patient mentioned no stress at the
workplace. Choice C usually presents with a chronic and progressive headache that is increasing in intensity.

You might also like